NOTEPAD
Results
of 200 questions answered correctly

You have reached of 200 points, ( %)

Your time

Question 1 of 200

1.

After   consumption  of   rich   food a patient has nausea  and heartburn, steatorrhea.   This   condition   might   be caused by:

Explanation

Obturation (obstruction, to close) of bile duct – it can be:

* Intrahepatic – blockage of intrahepatic bile ducts

* Extrahepatic – blockage of common bile duct (ductus choledochus).

Findings:

* malabsorption: bile salts do not enter the Small Intestine; no emulsification of fat.

*light coloured stool: due to lack of urobilin (which leads to lack of stercobilin).

*Jaundice (posthepatic, mechanic, obstructive): increased conjugated Bilirubin.

* Steatorrhea

The findings are specific for obstruction of bile duct and bile acid deficiency.

2.

An infant has apparent diarrhea resulting  from  improper  feeding.  One   of  the main  diarrhea effects  is plentiful  excretion of sodium  bicarbonate. What  form  of acid-base  balance  disorder is the case?

Explanation

Acidosis is the reduction in pH (increase in H+ concentration) below normal range. pH is less than 7.35; it is produced by:

·        Increase in partial pressure of CO2 in the body.

·        Decrease in HCO3- concentration.

Alkalosis is the increase in pH (decrease in H+ concentration) above normal range. pH is greater than 7.45; it is produced by:

·        Decrease in partial pressure of CO2 in the body.

·        Increase in HCO3- concentration.

Each of these two disorders has respiratory and non-respiratory forms. The non-respiratory form is divided into metabolic and excretory(non-gaseous).

·        Respiratory acidosis is the acidosis that is caused by alveolar hypoventilation e.g. airway obstruction due to bronchitis or lung diseases (pneumonia).

·        Respiratory alkalosis is caused by alveolar hyperventilation e.g. hypoxia in high altitude.

·        Non-respiratory:

-Metabolic acidosis is characterized by excess accumulation of organic acids such as lactic acid, ketoacids and uric acid formed by normal metabolism e.g. in Diabetes mellitus or extreme/prolonged exercise.

-Excretory/Non-gaseous acidosis may develop in impaired renal H+ excretion related to increased loss of bicarbonate in urine; diarrhea causes acidosis by the loss of bicarbonate with faeces.

-Excretory/Non-gaseous alkalosis: vomiting (loss of gastric acid), increased metabolism of lactate and citrate (turns into bicarbonate and water), long-term use of thiazides and loop diuretics.

      It is excretory/non-gaseous alkalosis because of the frequent vomiting.

3.

A  3  year  old  child  with  fever  was given   aspirin.   It   resulted  in   intensified erythrocyte haemolysis. Hemolytic anemia  might  have  been  caused  by congenital  insufficiency of the following enzyme:

Explanation

Idiosyncrasy and allergic reaction is as a result of organism’s increased sensitivity to the drugs. It’s a complication of pharmacological therapy (adverse reactions). 

 Idiosyncrasy is genetically conditioned perverse organism’s response to drugs (e.g. inefficiency, lack of enzymes participating in drug metabolism). Genetic deficiency of glucose 6-phosphate dehydrogenase (hereditary enzymopathy). This enzyme is needed for proper functioning of glutathione (an antioxidant) to prevent the oxidative effect of antimalarial drugs on the RBCs (erythrocytes).

4. A  patient  is  ill  with  diabetes  mellitus   accompanied  by  hyperglycemia  on an  empty  stomach  (7,2 millimole/l).  The hyperglycemia  rate   can  be  retrospectively  estimated (over  the  last  4-8 weeks before   the  examination) on  the  ground of the rate  of the following blood  plasma protein:

Explanation

One of the laboratory tests for diabetes mellitus is the level of glycosylated hemoglobin. Glycosylated hemoglobin test (HbA1C, glycohemoglobin) are proteins with glucose bound to them by nonenzymatic way. They precisely represent the extent of impairment of the carbohydrate metabolism and serve as the basic index of quality of compensation of diabetes mellitus. The level of HbA1C is determined by the method of chromatography using special laboratory equipment. Level of HbA1C shows an average blood concentration during the previous 2 - 3 months. Normal level of HbA1C is 4 – 6%. [4 – 8 weeks (1-2months)]

5.

In course of laparotomy a surgeon revealed  gangrenous lesion  of  descending colon. It was caused  by thrombosis of the following artery:

Explanation

The inferior mesenteric artery arises from the anterior aspect of the abdominal aorta. The artery gives the branches as follows: left colic artery (arteria colica sinistra); sigmoid arteries (supplies sigmoid colon) and superior rectal artery (supplies the rectum). Left colic artery (sinister colic) ascends leftwards to supply the descending colon. On reaching the intestine, the artery gives the ascending and descending branches that anastomose with the middle colic artery and sigmoid artery.

6.

A  37 year  old  patient suffering  from obliterating vascular endarteritis of lower limbs takes  daily 60 microgram/kilogram of   phenylin.   Because   of   presentations of convulsive disorder (craniocerebral trauma  in  anamnesis)  he   was  prescribed phenobarbital. Withholding this drug caused nasal hemorrhage. What is this complication connected with?

Explanation

        Phenylin is an anticoagulant. It has actions similar to warfarin, but it is now rarely employed because of its higher incidence of severe adverse effects. The barbiturate (Phenobarbital) is a sedative or used to induce and maintain sleep. But today, they have been largely replaced by the benzodiazepines (diazepam, nitrazepam), primarily because barbiturates induce tolerance, drug-metabolizing enzymes and physical dependence and are associated with very severe withdrawal symptoms. Barbiturates induce the CYP450 system and therefore, may decrease the duration of action of drugs that are metabolized by these hepatic enzymes. Through this mechanism, Phenobarbital causes a decrease in anticoagulant activity as measured by the prothrombin time. Phenobarbital induce hepatic microsomal enzymes resulting in increased metabolism and decreased anticoagulant response of oral anticoagulants (e.g. Phenylin, warfarin).

7.

A 60 year old patient has impaired perception   of   high-frequency   sounds. These changes were caused by damage  of the following auditory analyzer structures:

Explanation

Two membranes divide the spiral canal of cochlea into 3 compartments – vestibular and basilar membrane. Along the basilar membrane are 20,000-30,000 tiny fibers called basilar fibers. Each fiber has different size and shape. Fibers near the oval window (base) are short and stiff (narrowest), while approaching towards helicotrema (apex), the basilar fibers gradually become longer and soft (widest). The motion of the basilar membrane is generally described as a travelling wave. The parameters of the membrane (stiffness, narrow, wide) at a given point along its length determine its characteristic frequency at which it is most sensitive to sound vibrations. High frequency sounds localize near the base of the cochlea (near the round and oval windows), while low frequency sounds localize near the apex (helicotrema); and the middle frequency sounds – middle part of helix.

 

8.

Researchers   isolated    5   isoenzymic forms  of lactate  dehydrogenase from  the human blood serum and studied their properties. What  property indicates  that the  isoenzymic  forms  were  isolated  from the same enzyme?

Explanation

        Isoenzyme is any of several forms of an enzyme that all catalyze the same reaction but may differ in reaction rate, inhibition by various substances, electrophoretic mobility or immunologic properties. Examples of important enzymes with isoenzymic forms are:

·        Lactate dehydrogenase: LDH1, LDH2, LDH3, LDH4, LDH5, (5 isoenzymic forms).

·        Creatine kinase (CK): CK-MM; CK-MB; CK-BB (3 isoenzymic forms)

9.

It   was  proved   that   a  molecule   of immature mRNA (precursor mRNA) contained more  triplets  than  amino  acids   found    in   the   synthesized    protein. The  reason  for  that  is that  translation is normally preceded by:

Explanation

RNA molecules produced during transcription are functionally inactive. That is why they should undergo certain chemical modifications called processing or post-transcriptional modification of RNA. For eukaryotic mRNA processing includes 3 kinds of such alterations: 5’-capping; 3’-polyadenylation and splicing. Maturation of eukaryotic mRNA usually involves the removal of RNA sequences, which do not code for protein (introns, or intervening sequences) from the primary transcript. The remaining coding sequences, the exons, are joined together to form the mature mRNA. The process of removing introns and joining exons is called splicing. Abnormal splicing variants are implicated in oncogenesis and many genetic disorders (e.g. β-thalassemia).
10. A patient has low rate  of magnesium ions that  are  necessary  for affixion of ribosomes   to  the  endoplasmic  reticulum. It  is  known   that   it  causes  disturbance of protein biosynthesis. At what stage is protein biosynthesis impaired?

Explanation

A large number of components are required for translation (synthesis of a protein). These include all the amino acids that are found in the finished product, the mRNA to be translated, tRNA, functional ribosomes, energy sources and enzymes as well as protein factors needed for initiation, elongation and termination of the polypeptide chain. In eukaryotic cells, the ribosomes are either “free” in the cytosol or are in close association with the endoplasmic reticulum (which is then known as the “rough” endoplasmic reticulum or RER). The RER – associated ribosomes are responsible for synthesizing proteins that are to be exported from the cell as well as those that are destined to become integrated into plasma, endoplasmic reticulum or golgi membranes or incorporated into lysosomes.

11.

A  student takes  notes  of  a  lecture. Quality  of his notes  became  significantly worse when his neighbours began talking. What type of conditional reflex inhibition was the cause of it?

Explanation

The dog is conditioned to salivate at the sight of food and flash of light – that is a conditioned reflex that requires learning, memory and recall of previous experience (likewise the student, taking notes requires learning, memory and recall of previous experience). The established conditioned reflexes can be inhibited: externally and internally.

External inhibition: disturbing factors like a stranger, sudden noise (the students talking) or a strong smell can abolish the conditioned reflex and inhibit salivary secretion. This extra stimulus evokes the animal’s curiosity  and distracts the attention. Internal inhibition can be:

*Extinction of conditioned reflex: failure to reinforce the conditioned reflex by unconditioned stimulus.

*Differential: alteration of conditioned stimulus.

12.

A patient ill with neurodermatitis has been  taking prednisolone for a long time. Examination revealed  high rate  of sugar in his blood.  This complication is caused by the drug influence  upon  the  following link of carbohydrate metabolism:

Explanation

Glucocorticoids (prednisolone) stimulate gluconeogenesis. As a result, blood sugar rises, muscle protein is catabolized and insulin secretion is stimulated. Corticosteroids cause increased gluconeogenesis, increased lipolysis, CNS effects - at times including euphoria, maintenance of cardiovascular function by potentiation of norepinephrine. Gluconeogenesis uses the reversible reactions from glycolysis and 4 distinct reactions that circumvent the ones from glycolysis that are irreversible. These reactions are catalyzed by pyruvate carboxylase, phosphoenolpyruvate carboxykinase, fructose-1, 6-bisphosphatase and glucose-6-phosphatase. Glucagon, epinephrine and glucocorticoids stimulate all these enzymes (excluding pyruvate carboxylase).

13.

Labelled amino acids alanine  and tryptophane  were  injected   to  a  mouse in order  to  study  localization  of protein synthesis  in  its  cells.  The  labelled   amino acids will be accumulated near the following organellas:

Explanation

A large number of components are required for translation (synthesis of a protein). These include all the amino acids that are found in the finished product, the mRNA to be translated, tRNA, functional ribosomes, energy sources and enzymes as well as protein factors needed for initiation, elongation and termination of the polypeptide chain. In eukaryotic cells, the ribosomes are either “free” in the cytosol or are in close association with the endoplasmic reticulum (which is then known as the “rough” endoplasmic reticulum or RER). The RER – associated ribosomes are responsible for synthesizing proteins that are to be exported from the cell as well as those that are destined to become integrated into plasma, endoplasmic reticulum or golgi membranes or incorporated into lysosomes.

   The labeled amino acids will be accumulated close to the ribosomes and rough endoplasmic reticulum (RER).

14.

A patient with frequent attacks  of stenocardia  was  prescribed  sustak-forte to be taken  one tablet  twice a day. At first the effect was positive  but on the second day  stenocardia attacks   resumed.  What can explain  inefficiency of the prescribed drug?

 

Explanation

Idiosyncrasy and allergic reaction is as a result of organism’s increased sensitivity to the drugs. It’s a complication of pharmacological therapy (adverse reactions). Tolerance and Tachyphylaxis are effects of repeated doses.

* Idiosyncrasy is genetically conditioned perverse organism’s response to drugs (e.g. inefficiency, lack of enzymes participating in drug metabolism). Genetic deficiency of glucose 6-phosphate dehydrogenase (hereditary enzymopathy). This enzyme is needed for proper functioning of glutathione (an antioxidant) to prevent the oxidative effect of antimalarial drugs on the RBCs.

* Allergic reactions results from antibody production against some drugs.

* Tachyphylaxis is quick weakening of the effect, sometimes may occur after first drug introduction (usually connected with substrate exhaustion).

* Tolerance (habituation): is increasing of the effects in repeated doses resulting from decreasing of absorption, speed-up of the biotransformation or excretion, or decreasing of the receptors sensitivity.

15. Inflammation of the tympanic cavity (purulent otitis  media)  was  complicated by  inflammation of  mammillary   process sockets.   What   wall  of  tympanic   cavity did the pus penetrate into the sockets through?

Explanation

The walls of the tympanic cavity (6): tegmental wall, jugular wall, labyrinthine wall, membranous wall, carotid wall and mastoid wall. The mastoid wall (paries mastoideus) is the posterior wall related to the mastoid process of temporal bone. The opening on the wall – the aditus, to mastoid antrum leads to the greatest air cell called the mastoid antrum (antrum mastoideum) and further to smaller mastoid air cells.

16.

In  order   to  speed   up  healing  of  a wound of oral mucosa a patient was prescribed a drug  that  is a thermostable protein occurring  in tears,  saliva, mother’s milk as well as in a new-laid  hen’s egg. It is known  that  this  protein is a factor  of natural resistance  of an organism. What is it called?

Explanation

Innate or non-specific resistance consist of physical and chemical barriers such as skin, gastric acid, mucus or tears, saliva as well as cells and active mechanisms such as phagocytes, natural killer cells and the complement system. They defend an organism in non-specific form/manner, responding in the same fashion, regardless of what pathogen it is. Examples of physical barriers:

·        Skin: surface is made up of dead skin cells rich in keratin, which impedes microorganisms from entering the body – mechanical barrier

·        Gastric acid is a powerful defense against invading bacteria.

·        Saliva and tears contain antibacterial enzymes such as lysozyme, which destroy the cellular walls of bacteria.

·        Mucus is another defense; coating the mucus membranes and contains IgA antibodies (a component of specific/adaptive immune system).

17. An aged man had raise of arterial pressure under  a stress. It was caused  by activation of:

Explanation

     Stress activates the sympathoadrenal system and the hypothalamic-pituitary-adrenocortical (HPA) axis. Defense reactions involve catecholamine release, vagal withdrawal, cortisol secretion and activation of the rennin-angiotensin system. Catecholamine release is capable of increasing the arterial pressure. Sympathoadrenal system involves the sympathetic nervous system and the adrenal glands especially increased sympathetic activity that causes increased secretion of epinephrine by the adrenal medulla and norepinephrine by the postganglionic sympathetic nerve endings.

18.

A  20 year  old  patient complains  of general weakness, dizziness, quick fatigability.  Blood   analysis  results:  Hb-  80 g/l. Microscopical  examination results: erythrocytes are  of  modified  form.  This condition might be caused by:

Explanation

Sickle cell hemoglobin (HbS) point mutation causes a single amino acid replacement in β chain of hemoglobin (substitution of glutamic acid with valine). Pathogenesis of sickle cell anemia: low O2, high altitude or acidosis precipitates sickling – modified shape (deoxygenated HbS polymerizes) → anemia and vaso-occlusive disease. Clinical findings include: dizziness, general weakness, fatigue, “crew-cut” on skull X-ray due to marrow expansion from increased erythropoiesis (this is also seen in thalassemia)  etc. Sickle cells are crescent-shaped RBCs.

19.

Blood minute  volume of a 30 year old woman at rest is 5 l/m. What blood volume is pumped  through the pulmonary vessels per minute?

Explanation

Blood minute volume is the amount of blood pumped out by each ventricle in one minute. It is the product of stroke volume and heart rate. Normal value is 5L/ventricle/minute.

If blood minute volume is 5L/m; the volume pumped through the pulmonary vessels (lung vessels) per minute by the right ventricle equals 5L.

20. As a result of long-term starvation the glomerular filtration of a man was accelerated by 20%.  The  most  probable cause of filtration changes under such conditions is:

Explanation

       Starvation decreases protein synthesis because the substrates are not readily available. Decrease in protein synthesis decreases oncotic pressure and thereby increasing glomerular filtration. Net filtration pressure is the balance between hydrostatic pressure (glomerular capillary pressure) and oncotic pressure.

Net filtration pressure = Hydrostatic pressure – Oncotic pressure

Therefore, a decrease in oncotic pressure increases the filtration rate according to the degree of starvation which decreases protein synthesis.

21.

A patient has yellow skin colour, dark urine,  dark-yellow  feces. What  substance will have  strengthened concentration in the blood serum?

Explanation

 

Indirect; Hemolytic; Prehepatic

Mixed; Parenchymal; Hepatic

Direct; Obstructive; Mechanic; Posthepatic

Stercobilin (faeces)

        ↑↑↑

(dark urine, faeces)

Decreases (pale faces)

Absent (clay coloured faeces)

Type of bilirubin in blood

Unconjugated

Conjugated and Unconjugated

Conjugated

22.

A  patient consulted   a  doctor   about symmetric  dermatitis of open  skin areas. It  was  found  out  that  the  patient lived mostly on cereals  and ate too little meat, milk and eggs. What vitamin deficiency is the most evident?

Explanation

23.

A  46 year  old  patient applied   to  a doctor complaining about joint pain that becomes  stronger the day before  weather changes. Blood examination revealed strengthened concentration of  uric  acid. The  most  probable cause  of the  disease is the intensified  disintegration of the following substance:

Explanation

 The end product of the purine nucleotides catabolism in humans and other primates is uric acid (urate) which is excreted in urine. Allopurinol and febuxostat inhibits Xanthine oxidase (XO). Hypoxanthine and Xanthine which is more soluble is excreted in urine. Purine nucleotides (adenine and guanine).  AMP – Adenosine monophosphate; GMP – Guanosine monophosphate

Pyrimidine (thymine, Uracil, cytosine); catabolism of thymine and uracil gives urea, while catabolism of cytosine gives β-alanine.

 
24.

A  38  year  old  patient suffers  from rheumatism in its active phase. What laboratory characteristic of blood  serum is of diagnostic  importance in case of this pathology?

Explanation

     Acute phase reactants are factors whose serum concentrations change significantly in response to inflammation produced by the liver in both acute and chronic inflammatory states. Notably induced by IL-6 e.g. C-reactive protein, hepcidin, serum amyloid A are all upregulated; while albumin and transferring are downregulated. C-reactive protein is an opsonin, it fixes complement and facilitates phagocytosis. It is measured  clinically as a sign of ongoing inflammation.

25. ECG  of a patient with hyperfunction of thyroid  gland showed  heart  hurry.  It is  indicated by depression of the following ECG element:

Explanation

        Thyroid hormone increases synthesis of β-adrenergic receptors for catecholamines. Catecholamines (epinephrine) exert their effect on the heart by binding to β1-adrenoreceptors, which in turn increases the heart rate. Increase in heart rate shows a decrease in R-R interval on the ECG. This can lead to clinical manifestations such as systemic hypertension and high output heart failure.

26.

A     62    year     woman     complains   of   frequent   pain    attacks    in   the area   of   her   chest   and   backbone,  rib fractures. Her  doctor  suspected  myeloma (plasmocytoma). What of the following laboratory  characteristics will be  of  the greatest diagnostic importance?

Explanation

     Paraprotein, myeloma protein, M protein or spike protein is an abnormal immunoglobulin (Ig) fragment or immunoglobulin (Ig) light chain that is produced in excess by an abnormal clonal proliferation of plasma cells, typically in multiple myeloma. Monoclonal free light chains in the serum or urine are called bence jones (BJ) proteins.

Bence jones (BJ) protein: free kappa (κ) or lambda (λ) light chains that are excreted in urine associated with plasma cell malignancies (myeloma) and Waldenstrὄm macroglobulinemia. In myeloma, urinalysis for BJ protein is positive in 60-80% of cases.

Myeloma/Plasmocytoma/Multiple Myeloma

27.

While  playing  volleyball  a sportsman made  a jump  and  landed  on the  outside edge of his foot. He felt acute  pain in the talocrural joint,  active movements are  limited,  passive  movements are  unlimited but  painful.  A bit later  there  appeared a swelling in the area of external  ankle, the skin became  red and warm. What type of peripheral circulation   disturbance is the case?

Explanation

Hyperemia or active hyperemia or arterial hyperemia is the term used for the increased volume of blood in the arterial or arteriolar vessels. It is caused by an increased supply of blood from arterial system. The affected tissue or organ is pink or red in appearance (erythema) and warm to the touch.

     Venous or passive hyperemia also known as congestion; the affected tissue or organ is cold to the touch.

28.

Bacterioscopic examination of a smear from the  pharynx  of a diphtheria suspect revealed   bacilli  with  volutine   granules. What  etiotropic drug  should  be  chosen in this case?

Explanation

     Etiotropic drug deals directly with the etiological factor (diphtheria toxin). Antidiphtheric antitoxic serum neutralizes the toxin of diphtheria and is employed both as a curative and as a prophylactic agent. Diphtheric anatoxin (diphtheria toxoid) is the diphtheria toxin that has been weakened until it is no longer toxic but is strong enough to induce the formation of antibodies and immunity to the disease.

29. A  patient  underwent  an  operation on  account  of gall bladder excision  that resulted in obstruction of Ca2+ absorption through the  bowels  wall.  What  vitamin will stimulate  this process?

Explanation

Parathyroid hormone: secreted by chief cells of parathyroid gland. Effects include:

↑bone resorption of Ca2+ and PO43- → ↑their plasma levels

↑kidney reabsorption of Ca2+ in distal convoluted tubule → ↑ Ca2+ plasma level

↓reabsorption of PO43- in proximal convoluted tubule → ↓ PO43- plasma levels

↑Calcitriol (vit D3) production by stimulating kidney 1α-hydroxylase in proximal convoluted tubule. It increases Ca2+ and PO43- absorption in the intestine.

In general, parathyroid hormone ↑ Ca2+ plasma level but ↓ PO43- plasma levels. Abnormal synthesis (↑synthesis) of parathyroid hormone can lead to hypercalcemia and hypophosphatemia.

Calcitonin is secreted by parafollicular cells (C cells of the thyroid gland). It ↓bone resorption of Ca2+. It opposes actions of parathyroid hormone. But its not important in normal Ca2+ homeostasis. Calcitriol ↑ circulating Ca2+ ions as a means of enhancing intestinal absorption of calcium (NB: Calcitriol production is dependent on parathyroid hormone).
30.

A woman underwent an operation on account  of extrauterine (tubal) pregnancy. In  course  of  the  operation the  surgeon should  ligate the branches  of the following arteries:

Explanation

The uterine artery is one of the visceral branches of internal iliac artery. It descends on the lateral wall of the pelvis, anterior to the internal iliac artery and enters the root of the broad ligament. Here it passes medially to reach the lateral margin of the uterus immediately superior to the lateral part of the fornix of the vagina. As it passes medially in the broad ligament, the uterine artery passes directly superior to the ureter near the lateral part of the fornix of the vagina. On reaching the side of the cervix, the uterine artery divides into a smaller descending vaginal branch, which supplies the cervix and vagina and a larger ascending branch which runs along the lateral margin of the uterus, supplying the body and fundus of the uterus. The ascending branch bifurcates into ovarian and tubal branches which continue within the broad ligament to supply the medial ends of the ovary and uterine tube and anastomose with the ovarian and tubal branches of the Ovarian artery. Tubal pregnancy refers to pregnancy in uterine tube.
31.

A 6 month  old baby ill with bronchitis was taken  for an X-ray of chest. Apart of changes associated with bronchi the X-ray film showed  a shadow  of  thymus  gland. What might have caused such changes?

Explanation

There is a certain correlation between age of an individual and thymus activity. In neonates, it already appears to have a considerable mass of 13.3g on the average. Most intensive growth occurs during first 3 years of life, when the gland doubles in size. Having reached the maximum weight (about 26 – 30g), the thymus retains it until 20years of life. After 20 years of age, the thymus parenchyma experiences gradual involution and is substituted with fat tissue. After 50 years of age, the fat constitutes 90% of gland mass.

Age involution: therefore, it is normal to see a shadow of thymus gland in a 6 months old child.
32.

A  patient complains  of  frequent diarrheas, especially  after  consumption of rich food, weight loss. Laboratory examination    revealed    steatorrhea;  his   feces were hypocholic. What might have caused such condition?

Explanation

Obturation (obstruction, to close) of bile duct – it can be:

* Intrahepatic – blockage of intrahepatic bile ducts

* Extrahepatic – blockage of common bile duct (ductus choledochus).

Findings:

* malabsorption: bile salts do not enter the Small Intestine; no emulsification of fat.

*light coloured stool: due to lack of urobilin (which leads to lack of stercobilin).

*Jaundice (posthepatic, mechanic, obstructive): increased conjugated Bilirubin.

* Steatorrhea

The findings are specific for obstruction of bile duct and bile acid deficiency.

33.

Examination of a patient with pustular skin lesions  allowed  to isolate  a causative agent that forms in the blood agar roundish yellow middle-sized  colonies surrounded by  haemolysis  zone.  Smears from the colonies contain irregular- shaped   clusters   of  gram-positive  cocci. The   culture   is  oxidase-   and   catalase- positive,  ferments mannitol and synthesizes plasmocoagulase.   What    causative agent was isolated?

Explanation

Staphylococcus aureus is a gram positive cocci that appear in clusters. It causes:

·        Inflammatory disease  like skin infections, organ abscesses

·        Toxin mediated disease like the toxic shock syndrome (TSST-1)

·        MRSA (methicillin-resistant Staphylococcus aureus) infection.

Staphylococcus aureus is a β-hemolytic bacteria i.e. they form a clear zone of hemolysis on blood agar. They are catalase and coagulase (plasmocoagulase) positive.

34.

A 65 year old man suffering from gout complains of kidney pain. Ultrasound examination revealed   renal  calculi.  The most probable cause of calculi formation is the  strengthened concentration of the following substance:

Explanation

The end product of the purine nucleotides catabolism in humans and other primates is uric acid (urate) which is excreted in urine and can also cause kidney stones (renal calculi). Allopurinol and febuxostat inhibits Xanthine oxidase (XO). Hypoxanthine and Xanthine which is more soluble is excreted in urine. Purine nucleotides (adenine and guanine).  AMP – Adenosine monophosphate; GMP – Guanosine monophosphate

Pyrimidine (thymine, Uracil, cytosine); catabolism of thymine and uracil gives urea, while catabolism of cytosine gives β-alanine.

35. Microscopic  examination of a Gram- stained  scrape  from  patient’s  tongue revealed  oval, round,  elongated chains of dark-violet gemmating cells. What disease can be caused by this causative agent?

Explanation

       There are two types of fungi: yeasts and molds. Yeasts grow as single cells that reproduce by asexual budding (gemmating). Molds grow as long filaments (hyphae) and form a mat (mycelium). Candida albicans is an oval yeast with a single bud. It is part of the normal flora of mouth (oral cavity), mucous membranes of upper respiratory, gastrointestinal and female genital tracts. In tissues it may appear as yeast or as pseudohyphae. Pseudohyphae are elongated yeasts (elongated chains) that visually resemble hyphae but are not true hyphae. Candida albicans causes thrush, vaginitis, esophagitis and chronic mucocutaneous candidiasis.

36. A  44 year  old  woman  complains  of general weakness, heart pain, significant increase   of   body   weight.   Objectively: moon  face, hirsutism,  AP  is 165/100 mm Hg, height  - 164 cm, weight - 103 kg; the fat  is mostly  accumulated on  her  neck, thoracic   girdle,  belly.  What  is the  main pathogenetic mechanism  of obesity?

Explanation

Cushing Syndrome: Etiology

*INCREASE Cortisol due to a variety of causes (Glucocorticoids).

*Exogenous corticosteroids:  result in DECREASE ACTH (MCC).

*Primary adrenal adenoma, hyperplasia or carcinoma (Cushing’s Syndrome).                                     *ACTH-secreting pituitary adenoma (Cushing Disease).

Findings: Hypertension, Weight Gain, Moon Facies, Truncal Obesity, Buffalo Hump, Skin Changes (thinning striae), Osteoporosis, Hyperglycemia (Insulin resistance), Amenorrhea, Immunosuppression.

37.

Blood  of  a  patient with  presumable sepsis was inoculated into sugar broth. There  appeared bottom  sediment. Repeated inoculation into  blood  agar caused growth of small transparent round colonies  surrounded by  hemolysis  zone. Examination of  a  smear  from  the  sediment revealed  gram-positive cocci in form of long chains. What  microorganisms are present in blood of this patient?

Explanation

           There are two medically important genera of gram positive cocci: Staphylococcus and Streptococcus. They are nonmotile and do not form spores. Microscopically, staphylococci appear in grape-like clusters and produce catalase (i.e. catalase positive), whereas streptococci are in chains and do not produce catalase (i.e. catalase negative). One of the most important characteristics for identification of streptococci is the type of hemolysis they manifest (alpha-hemolytic and beta hemolytic). Alpha-hemolytic (α-hemolytic) streptococci form a green zone around their colonies as a result of incomplete lysis of RBCs in the agar. Beta-hemolytic (β-hemolytic) streptococci form a clear zone around their colonies because complete lysis of the RBCs occurs. Β-hemolysis is due to the production of enzymes (hemolysins) called streptolysin O and streptolysin S. Some streptococci are non-hemolytic (gamma-hemolysis).

38.

2 years ago a patient underwent resection of pyloric  part  of stomach.  He complains of weakness, periodical dark shadows beneath his eyes, dyspnea. In blood:  Hb  - 70 g/l, erythrocytes - 3, 0 ·1012/l,  colour  index  - 0,7. What  changes of  erythrocytes in blood  smears  are  the most typical for this condition?

 

Explanation

       Iron requires an acidic medium in stomach to be absorbed. Iron is needed in heme synthesis (hemoglobin). A deficiency of iron will produce marked hypochromia (Hb – 70g/L; Normal: 120-140 female; 135-165 male) and decreased colour index (normal 0.85 – 1.05).

Megalocytes and macrocytes are usually caused by Vitamin B12 and Folic acid deficiencies which often produce normochromic/hypochromic anemia because they are not really involved in hemoglobin synthesis. The resection of the pyloric part of stomach can reduce the gastric acid level thereby reducing iron absorption.

39.

A  patient is ill with  hepatocirrhosis. State of antitoxic liver function can be characterized   by   examination   of   the following substance  excreted by urine:

Explanation

Hippuric acid has been a major human metabolite for years. However, there is no well-known documented health benefit associated with it except for excretion of environmental-toxic exposures of aromatic compounds such as toluene or from dietary protein degradation and resynthesis by intestinal microflora metabolism of quinic acid via the shikimate pathway. Thus hippuric acid can appear in humans as an excretory product from natural or unnatural sources. It has been believed over the years that the major source of urinary hippuric acid levels in humans has come from environmental toxic solvent exposures.

40.

The  territory of an old burial  ground for animal refuse that hasn’t been used for over 50 years is meant  for house building. But soil investigation showed the presence of viable spores of a causative agent causing  a  very  dangerous disease.  What  microorganism might  have  been  preserved in soil for such a long period  of time?

Explanation

There are two medically important Bacillus species: Bacillus anthracis and Bacillus cereus. Bacillus anthracis causes anthrax. Human disease occurs in 3 main forms: cutaneous, pulmonary (inhalation) and gastrointestinal. Humans are most often infected cutaneously at the time of trauma to the skin, which allows the spores on animal products such as hides, bristles and wool to enter. Spores can also be inhaled or when contaminated meat is ingested. After being inhaled, the organism moves rapidly to the mediastinal lymph nodes and causes hemorrhagic mediastinitis. Pathogenesis is based on the production of two exotoxins  (Anthrax toxin) – edema factor and lethal factor. Hemorrhagic mediastinitis, septic shock hemorrhagic meningitis and death are severe life-threatening complications. In fatal cases, the organism may affect the spleen, liver, intestines, kidneys, adrenal glands and meninges.

41.

Autopsy  of  a  man   who  had   been working as a miner for many years and died from  cardiopulmonary decompensation   revealed    that    his   lungs   were   airless, sclerosed, their apexex had emphysematous changes, the lung surface was  greyish-black,   the   incised   lung  tissue was coal-black.  What  disease caused death?

Explanation

Anthracosis also known as Coal workers’ pneumoconiosis or black lung disease; it is caused by lung exposure to coal dust. It is common in coal miners and others who work with coal. Inhaled coal dust progressively builds up in the lungs and is unable to be removed by the body; this leads to inflammation, fibrosis and in worse cases necrosis.

42.

Microscopy  of stained  (Ziehl-Neelsen staining)  smears  taken  from  the  sputum of  a patient with  chronic  pulmonary disease revealed  red bacilli. What  property of tuberculous bacillus was shown up?

Explanation

           Mycobacteria are aerobic, acid fast bacilli (rods). They are neither gram +ve nor  gram -ve i.e. they are stained poorly by the dyes used in gram stain. They are virtually the only bacteria that are acid-fast (one exception is Nocardia asteroids, the major cause of Nocardiosis, which is also acid-fast). The term  “acid-fast” refers to an organism’s ability to retain the carbolfuchsin stain despite subsequent treatment with an ethanol-hydrochloric acid mixture. The high lipid content (approx. 60%) of their cell wall makes Mycobacteria acid-fast.The major pathogens are Mycobacterium tuberculosis, the cause of tuberculosis and Myobacterium lepra, the cause of leprosy. M. tuberculosis grows slowly. Its cell wall contains long chain (C78-C90) fatty acids called mycolic acids, which contribute to the organism’s acid-fastness. Ziehl-Neelsen stain is also known as acid-fast stain.

43.

Parents of a 10 year old boy consulted a doctor  about  extension  of hair-covering, growth  of beard  and moustache, low voice. Intensified secretion of which hormone must be assumed?

Explanation

        The clinical manifestations are secondary sexual characteristics of an adult male.

Early in male development, mesenchyme separating the seminiferous cords gives rise to leydig (interstitial) cells that produce testosterone to stimulate development of the indifferent primordium into a testis. Leydig cells are large, polygonal, eosinophilic cells that typically contain lipid droplets. Leydig cells differentiate and secrete testosterone during early fetal life. Secretion of testosterone is required during embryonic development, sexual maturation and reproductive function:

·        In the embryo, secretion of testosterone and other androgens is essential for the normal development of the gonads in the male fetus.

·        At puberty, secretion of testosterone is responsible for the initiation of sperm production, accessory sex gland secretion and development of secondary sex characteristics.

·        In the adult, secretion of testosterone is essential for the maintenance of spermatogenesis and of secondary sex characteristics, genital excurrent ducts and accessory sex glands.

 The manifestation of these characteristics in a 10year old boy is as a result of increased secretion of the male sex hormone – Testosterone.

44.

Examination of coronary arteries revealed  atherosclerotic calcified plaques closing  vessel  lumen  by 1/3. The  muscle has multiple  whitish layers  of connective tissue.  What  process  was revealed  in the myocardium?

Explanation

Diffuse cardiosclerosis

Diffuse – multiple whitish layers of connective tissue

Cardiosclerosis – Atherosclerotic calcified plaques

. Focal cardiosclerosis. Focal –white fibrous depressed area 3cm in diameter.    Cardiosclerosis-Atherosclerosis.     
45.

It  was  revealed   that   T-lymphocytes were  affected   by  HIV. Virus  enzyme - reverse    transcriptase (RNA-dependent DNA polymerase)  -  catalyzes the synthesis of:

Explanation

RNA dependent DNA polymerase

     DNA polymerase – synthesizes DNA

     RNA dependent – from mRNA (on the matrix of virus mRNA)

       Human immunodeficiency virus (HIV): diploid genome (2 molecules of RNA). The 3 structural genes(i.e. proteins coded for by the genes) are:

·        env (gp 120 and gp 41): formed from cleavage of gp 160 to form envelope glycoproteins. gp 120 is for attachment to host CD4+ T cell. gp 41 is for fusion and entry.

·        Gag (p24): capsid protein

·        pol: reverse transcriptase, aspartate protease, integrase.

ELISA/Western blot (immunoblot) tests look for antibodies to the viral proteins listed above.

Reverse transcriptase synthesizes dsDNA (ds-double stranded) from genomic RNA (mRNA); dsDNA integrates into host genome. Virus binds CD4 as well as a coreceptor, either CCR5 on macrophages (early infection) or CXCR4 on I cells (late infection).

·        Homozygous CCR5 mutation – immunity

·        Heterozygous CCR5 mutation – slower course.

46. A  35  year  old  man  with  a  trauma of his left hand was admitted to the traumatology  department.   Objectively: cut wound of palmar  surface of left hand; middle  phalanxes  of  II–V  fingers  don’t bend. What muscles are damaged?

Explanation

   Flexor digitorum superficialis (superficial finger flexor) arises by two heads – the humero-ulnar head and radial head from the corresponding bones. The spindle-shaped belly of the muscle divides into 4 long tendons, which pass under the flexor retinaculum and attach to the bases of the middle phalanges of the II through V digits. Prior to this, each tendon separates into two slips and forms a cleft, which gives passage to the tendon of the flexor digitorum profundus. The flexor digitorum profundus (profound/deep finger flexor) resides in the middle of the deep layer underneath the flexor digitorum superficialis. So, a trauma of the hand will affect the superficial muscles first before the deep (profundus) muscles are affected. Therefore, superficial finger flexor is the more correct answer.

47.

Histological   examination  of  a  skin tissue sampling revealed granulomas consisting of macrophagal nodules with lymphocytes   and  plasmatic   cells.  There are also some big macrophages with fatty vacuoles  containing   causative   agents  of  a  disease  packed  up  in form  of  spheres (Virchow’s  cells).  Granulation  tissue   is well vascularized.  What disease is this granuloma typical for?

Explanation

       Leprosy (Hansen’s disease) is a chronic generalized infectious disease, which involves mainly the ectoderm components (the skin, mucouc membranes and the peripheral nervous system), lymphatic nodes, liver, spleen, bones and endocrine glands. The causative agent is acid proof and alcohol-resistant Mycobacteria leprae. Three types of leprosy are distinguished: lepromatous; tuberculoid; undifferentiated (the first 2 are the main forms). In tuberculoid leprosy, the epidermis contains confluent granulomas composed of macrophages, plasma cells and leprous Virchow’s cells. Leprous Virchow’s cells (or leprosy/hansen’s cells) refer to large foamy macrophages within fatty vacuoles containing leprous mycobacterium.

48.

In order  to estimate toxigenity  of diphtheria  agents   obtained  from  patients the cultures  were inoculated on Petri dish with nutrient agar on either  side of a filter paper  strip  that  was put  into  the  centre and  moistened with  antidiphtheric anti-toxic serum. After incubation of inoculations in agar the strip-like areas of medium turbidity  were found between separate cultures  and the strip of filter paper.  What immunological reaction was conducted?

 

Explanation

        Precipitation in agar(Precipitation gel reaction): antibody is incorporated into agar and antigen is inoculated i.e. antibody is the antidiphtheric antitoxic serum and the antigen is the diphtheria agent, inoculated in the nutrient agar. Antigen and antibody are placed in different wells in agar and allowed to diffuse and form concentration gradients. Where optimal proportions occur, lines of precipitate form. This method indicates whether antigens are identical, related but not identical or not related.

Areas of medium turbidity indicate where the antibody specific for the antigen have cross-linked.

49. A patient taking clonidine  for essential hypertension treatment was using alcohol that caused intense inhibition of central  nervous  system.  What  mait be connected with?

Explanation

50.

Examination of an ovary specimen stained   by  hematoxylin-eosine  revealed a follicle in which follicular epithelium consisted of 1-2 layers of cubic cells. There was also  a bright  red  membrane around the ovocyte. What follicle is it?

Explanation

At puberty, a pool of growing follicles is established and continuously maintained from the supply of primordial follicles. A primary oocyte, together with its surrounding flat epithelial cells forms the primordial follicle. Each month 15 – 20 follicles selected from this pool begin to mature, passing through 3 stages namely: primary or preantral; secondary or antral and preovulatory (graafian follicle). As the primary oocyte begins to grow, surrounding follicular cells change from flat to cuboidal and proliferate to produce a stratified epithelium (which can be 1 - 2 layers but usually more) of granulose cells and the unit is called a primary follicle.

51.

A patient with clinical signs of encephalitis was delivered to the  infectious diseases hospital. Anamnesis registers a tick bite. Hemagglutination-inhibition reaction helped to reveal antibodies to the causative  agent of tick-borne encephalitis in the dilution 1:20 which is not diagnostic. What actions should the doctor  take after he had got such result?

 

Explanation

Tick-borne encephalitis is a disease caused by a flavivirus that affects the central nervous system. It is transmitted through the bite of an infected tick. A serum sample is obtained as soon as a viral etiology is suspected (acute phase) and a second sample is obtained 10-14days later (convalescent phase). If the antibody titer in the convalescent phase serum sample is at least fourfold higher than the titer in the acute phase serum sample, the patient is considered to be infected. For example, if the titer in the acute phase serum sample is ¼ and the titer in the convalescent phase serum sample is 1/16 or greater, the patient has had a significant rise in antibody titer and has been recently infected. If however, the titer in the convalescent phase serum is 1/8, this is not a significant rise (not diagnostic) and should not be interpreted as a sign of recent infection, but a second sample is obtained 10-14days later.

52.

A 40 year old man noticed  a reddening  and  an  edema   of  skin  in  the   area of  his  neck  that  later  developed into  a small abscess. The  incised focus is dense, yellowish-green.  The  pus  contains   white granules. Histological examination revealed drusen of a fungus, plasmatic and xanthome cells, macrophages. What  type of mycosis is the most probable?

Explanation

Actinomycetes are true bacteria (related to Corynebacteria and Mycobacteria), but they form long, branching filaments that resemble the hyphae of fungi. They are gram positive but some (such as Nocardia asteroids) are also weakly acid-fast. There are two medically important organisms, Actinomyces israelii and Nocardia asteroids. Actinomyces israelii causes Actinomycosis (abscess with draining sinuses). Strictly anaerobic; it forms part of the normal flora of the oral cavity. Actinomycosis appears as a hard, nontender swelling that develops slowly and eventually drains pus through sinus tracts. In about 50% of cases, the initial lesion involves the face and neck.

53.

Continuous taking of some drugs foregoing  the  pregnancy  increase  the  risk of giving birth  to a child with genetic defects. What is this effect called?

Explanation

54.

A man with a wound  of his limb that had been  suppurating for a long time died  from  intoxication. Autopsy  revealed extreme emaciation, dehydration, brown atrophy of liver, myocardium, spleen  and cross-striated muscles as well as renal amyloidosis. What diagnosis corresponds with the described  picture?

 

Explanation

        Chronic sepsis (chroniosepsis): this form of sepsis is characterized by durably availability, not healing of primary septic focus that progress into a chronic one. In organs and tissues, there is atrophy and dehydration are expressed. Brown atrophy is found in the liver, myocardium and striated muscles. The spleen is decreased.

55.

6  months    after   labour    a   woman had   uterine  hemorrhage.  Gynaecological examination of uterine cavity revealed a  dark-red tissue  with  multiple   cavities resembling of a \\\"sponge\\\". Microscopic examination  of   a   tumour  revealed   in blood lacunas atypic light epithelial Langhans  cells and  giant cells of syncytiotrophoblast. What tumour is it?

Explanation

Chorioepithelioma or choriocarcinoma or gestational choriocarcinoma is a malignant neoplasm of trophoblastic cells derived from a previously normal or abnormal pregnancy. NB: trophoblast differentiates into 2 layers (cytotrophoblast and syncytiotrophoblast) in the 2nd week of development of an embryo. Chorioepithelioma is a soft, fleshy, yellow-white tumor that usually has large pale areas of necrosis and extensive hemorrhages. Histologically, it does not produce chorionic villi and consists entirely of proliferating syncytiotrophoblasts and cytotrophoblasts.

56.

An  animal  with  aortic  valve  insufficiency    got    hypertrophy   of    its    left heart   ventricle.   Some  of  its  parts  have local contractures. What substance accumulated  in  the  myocardiocytes caused these contractures?

Explanation

 Increase in cardiac workload can cause imbalances in oxygen (O2) supply and demand to the heart itself. Blood supply to the heart occurs during diastole. Increase heart rate (↑HR) → ↓diastolic period → ischemia in myocardium. Ischemia, hypoxia, heart failure are associated with disruptions in intracellular Na+ and Ca+ concentration homeostasis of myocardial cells. A decrease efflux or increase influx of Na+ may cause cellular Na+ overload. Na+ overload is followed by an ↑influx of calcium (Ca2+) through Na+-Ca2+ exchanger. Failure to maintain the homeostasis of Na+ and Ca+ leads to electrical instability (arrhythmias), mechanical dysfunction (reduced contractility and ↑ diastolic tension) and mitochondrial dysfunction. These events increase ATP hydrolysis and decrease ATP formation and if left uncorrected, they cause cell injury and death → acute heart failure.

57.

A  girl is diagnosed with adrenogenital syndrome (pseudohermaphroditism). This pathology was caused by hypersecretion of  the  following  adrenal hormone:

Explanation

Adrenogenital syndrome results from the hereditary stipulated blockade of cortisol synthesis and amplified formation of androgens from general intermediate products.
58.

In  course   of  an  experiment  a  big number  of stem cells of red bone  marrow was in some way destructed. Regeneration of which cell populations in the  loose connective tissue will be inhibited?

Explanation

Monocytes are the precursors of the cells of the mononuclear phagocytic system. Monocytes are the largest of the white blood cells in a blood smear. They travel from the bone marrow to the body tissues, where they differentiate into the various phagocytes of the mononuclear phagocytic system i.e. connective tissue macrophages (histiocytes), osteoclasts, alveolar macrophages, perisinusoidal macrophages in the liver (kupffer cells) and macrophages of lymph nodes, spleen and bone marrow, among others. Monocytes remain in the blood for only about 3 days. Monocytes transform into macrophages, which function as antigen presenting cells in the immune system.

59.

Examination  of  a  70 year  old  patient revealed insulin-dependent diabetes. What drug should be administered?

Explanation

Sulfonylurea are insulin secretagogues because they promote insulin release from the β-cells of the pancreas. The primary drugs used today are the 2nd generation drugs – glibenclamide, glipizide, glimepiride etc. These agents are useful in the treatment of patients who have type 2 diabetes mellitus and cannot be managed by diet alone. Metformin is the only currently available biguanide; its classed as an insulin sensitizer. Acarbose – α glucosidase inhibitor. Butamide (tolbutamide) is also a sulfonylurea but is older and glibenclamide is more potent and used much more commonly. Actrapid (soluble insulin) is fast acting but not a sulfonylurea.

60. A  human  body  cools  in water  much faster  that  in the  air.  What  way of heat emission in water is much more efficient?

Explanation

Conduction is a way the body eliminates heat by means of direct contact with another object. Heat is transferred down the temperature gradient (i.e. from the object of higher temperature to the object of lower temperature). Conduction requires contact with another object (Key words:  in water).

Heat Radiation is a way the surface of the human body emits heat to the environment in the form of infrared rays. The amount of heat the body radiates to the environment is proportional to the surface of radiation area and to the difference between the mean values of skin and environment temperature. The surface radiation area is the total surface area of body parts that contact the air. Elimination of heat by radiation increases with a decrease in ambient temperature and decreases with its increase. It is possible to reduce elimination of heat by radiation via reduction of the surface of radiation area (“winding oneself into a ball”). Heat radiation does not require a medium for transfer of heat. (Key words: naked or lightly clothed).

Convection is a way the body eliminates heat by means of transferring heat via moving particles of air or water. To dissipate heat by means of convection, body surface shall be airflowed at a temperature that is lower than the temperature of the skin. At that, air layer contacting with the skin warms up, decreases its density, rises and is replaced by cooler, denser air. By increasing the speed of the air flow (wind, ventilation) heat emission increases significantly as well (forced convection). Convection requires convection current; current of gases or liquids (Key words: air over exposed area of skin).

Evaporation is a way the body dissipates heat to the environment by its evaporation via sweat or evaporation of moisture from the skin and respiratory tract mucous membranes of (“wet” heat loss). Evaporation closely related to relative humidity.

 

61.

A   patient  suffering   from   chronic cardiac   insufficiency  was  recommended to undergo a prophylactic course  of treatment with a cardiological  drug  from the  group   of  cardiac   glycosides  that   is to be taken  enterally. What drug was recommended?

Explanation

Cardiac glycoside is the most appropriate in this case. Cardiac glycosides are glycosidic drugs of plant origin having cardiac inotropic property, increase myocardial contractility and output.

Corglyconum is short acting for acute cardiac insufficiency and administered intravenously.

Digitoxin is long-acting but basically administered orally and used mainly for chronic heart failure (also digoxin).

Cordiamin – analeptic; Adrenaline is commonly used in shocks and coma; 

62.

An  ophthalmologist used  a 1% mesaton  solution  for  the  diagnostic purpose (pupil dilation  for eye-ground examination). What is the cause of mydriasis induced  by the drug?

Explanation

1     

63.

Prophylactic medical  examination of a  36  year  old  driver   revealed   that   his AP  was  150/90 mm  Hg.  At  the  end  of workin day  he  usually  hear ear  noise,  feels  slight  indisposition  that  passes after  some  rest.  He  was diagnosed with essential  hypertension. What  is the leading pathogenetic mechanism  in this case?

Explanation

64. A  cerebral trauma caused  increased ammonia  generation. What amino acid participates in the  excretion of ammonia from the cerebral tissue?

Explanation

Substances absorbed into the bloodstream from the intestine pass through the liver, where toxins are normally removed. Many of these toxins (such as ammonia) are normal breakdown products of the digestion of protein.  Ammonia is produced by amino acid metabolism and intestinal urease-positive bacteria. In physiological conditions, it is mostly present as ammonium (NH4+) in serum. The urea or ornithine cycle, which is fully expressed in the liver exclusively, serves to converts NH4+ to urea prior to renal excretion and to maintain low serum concentrations. 

NH3 + α-ketoglutarate → Glutamate

α-ketoglutarate is used up which leads to:

·        ↑glutamate → ↑GABA (inhibitory neurotransmitter)

·        Inhibition of citric acid cycle/tricarboxylic acid cycle; this causes impairment of ATP formation.

·        Inhibition of metabolism of amino acids (impairment of transamination reactions).

 

NH3 + Glutamate → Glutamine

Glutamine is an amide of glutamic acid which provides a non-toxic storage and transport form of ammonia (NH3). Ammonia increase synthesis of glutamine in brain. Accumulation of glutamine in brain results in elevation of osmotic pressure in nervous cells leading to brain edema.

                                   

                                    NH3 + H+ → NH4+

In blood ammonia (NH3) is represented as ammonium ion (NH4+). Accumulation of ammonium ion impairs transport of ions (Na+, K+) through cell membranes and failure of transmission of nerve impulse.

            Urea cycle takes place exclusively in the liver, so in hepatic coma, urea level is low. Glutamine toxicity in brain is dependent on increased ammonia concentration.

Bilirubin toxicity will most likely be related to increase hemolysis, which is not the case in this question. Histamine is a biogenic amine produced from the amino acid histidine.

65.

While  studying  maximally  spiralized chromosomes of human karyotype the process of cell division was stopped in the following phase:

 

Explanation

Gametogenesis involves the generation of germ cells (gametes) through meiosis. Meiosis requires 2 cell divisions, meiosis I and II, in order to reduce the number of chromosomes to the haploid number of 23. At the beginning of meiosis I, germ cells replicate their DNA so that each of the 46 chromosomes is duplicated into sister chromatids. Homologous pairs then separate into two daughter cells, thereby reducing the chromosome number from diploid to haploid. Shortly thereafter, meiosis II separates sister chromatids (2 sister chromatids are joined in the middle by a centromere). Each gamete then conatins 23 chromosomes.

Metaphase is a stage of cell division in which chromosomes are at their most condensed and coiled stage and chromosomes are arranged along the equatorial plane during this phase. Cell division is usually studied during this phase.

66. 48   hours    after    tuberculine   test (Mantoux test)  a child  had  a  papule  10 mm in diameter on the spot of tuberculine injection.  What hypersensitivity mechanism underlies these changes?

Explanation

Type IV (cell mediated, delayed, cellular cytotoxicity): antibody-independent T-cell mediated reactions e.g. positive mantoux reaction (tuberculin test), hashimoto’s thyroiditis or transplant rejection etc.

Mantoux test is a type IV Hypersensitivity reaction (HSR), which involves macrophages,T-lymphocytes and lymphokines(cytokines). Mononuclear cells (lymphocytes,monocytes,macrophages).

Remember,it is antibody independent (i.e does not involve antibodies).

B-lymphocytesàPlasma cellsàIg(Antibodies)------- none is involved in Tyoe IV HSR.

The Mantoux skin test should be read between 48 and 72hrs after administration. The basis of reading  is the presence or absence of induration, which may be determined by inspection and by palpation. A record should also be made of formation of vesicles, bullae, lymphangitis, ulceration and necrosis at the test site. The formation of vesicles, bullae or necrosis at the test site indicates positive result. A negative mantoux result usually signifies that the individual has never been exposed to Mycobacterium tuberculosis i.e. absence of cell mediated immunity to tuberculin.

67.

A patient was admitted to the surgical department with suspected  inflammation of Meckel’s diverticulum. What part of bowels  should  be  examined in  order  to discover  the diverticulum in course  of an operation?

Explanation

Ileal diverticulum or Meckel’s diverticulum is a congenital anomaly that occurs in 1-2% of the population. A remnant of the proximal part of the embryonic omphaloenteric duct (yolk stalk), the diverticulum usually appears as a finger-like pouch. It is always at the site of attachment of the omphaloenteric duct on the antimesenteric border (border opposite the mesenteric attachment) of the ileum. The diverticulum is usually located 30-60cm from the ileocecal junction in infants and 50cm in adults. It may be free (74%) or attached to the umbilicus (26%). Although its mucosa is mostly ileal in type, it may also include areas of acid-producing gastric tissue, pancreatic tissue or jejuna or colonic mucosa. An ileal diverticulum may become inflamed and produce pain mimicking that produced by appendicitis.

68. Power inputs of a man were measured. In  what  state  was this  man  if his power inputs were lower than basal metabolism?

Explanation

        Sleep is the natural periodic state of rest for mind and body with closed eyes characterized by partial or complete loss of consciousness. Sleep is unconsciousness from which the person can be aroused by sensory or other stimuli. It is to be distinguished from coma, which is unconsciousness from which the person cannot be aroused. During sleep, most of the body functions are reduced to basal level or even below the basal level. Therefore, power inputs can be lower than basal level during sleep. Unlike rest where it remains at the basal level or higher.

69.

Examination of a newborn  boy’s genitalia   revealed   an   urethral  hiatus   that opens on the undersite of his penis. What malformation is it?

Explanation

Defects in male genitalia:

Hypospadia: fusion of the urethral folds is incomplete and abnormal openings of the urethra occur along the inferior (undersite) surface of the penis, usually near the glans, along the shaft or near the base of the penis.

Epispadia is a rare abnormality (1/30000 births) in which the urethral meatus is found on the dorsum (superior) surface of the penis.

70. A concentrated solution  of sodium chloride  was intravenously injected  to an animal. This caused decreased reabsorption of sodium  ions in the renal  tubules.  It is the  result  of the  following  changes  of hormonal secretion:

Explanation

Aldosterone produced in adrenal cortex (zona glomerulosa): causes increased sodium (Na+) reabsorption; increased potassium and hydrogen ions (↑K+, H+) excretion. They increase sodium (↑Na+) channel and Na+ /K+-pump insertion in principal cells of collecting duct; enhances K+ and H+ excretion by way of principal cell K+ channels and α-intercalated cell H+ ATPases of collecting duct. Therefore, increase in aldosterone → ↑ K+ in urine (excretion) and ↓ Na+ in urine (↑ reabsorption); And decreased aldosterone → ↓ K+ excretion (↓K+ in urine) and ↓ Na+  reabsorption (i.e. ↑Na+ in urine); same effects on sweats glands too.

71. As  a result  of destruction of certain brainstem structures an  animal  has  lost its  orientative  reflexes   in  response   to strong   photic   stimuli.   What   structures were destroyed?

Explanation

        Quadritubercular bodies/tectal plate is the dorsal portion of the midbrain comprises 4 colliculi – two superior and two inferior. The nuclei of colliculi are responsible for reflexes associated with sudden sound and visual stimuli (auditory and visual orientative reflexes); they also maintain consciousness. The nuclei give rise to the tectospinal tract. Superior or Anterior colliculi – visual orientative reflex; Inferior or Posterior colliculi – auditory orientative reflex.

72. Myocyte cytoplasm  contains  a big number  of  dissolved  metabolites of glucose oxidation.  Name one of them that turns directly into a lactate:

Explanation

Lactate, formed by the action of lactate dehydrogenase (converting pyruvate to lactate) is the final product of anaerobic glycolysis in eukaryotic cells. In organs or cells that are poorly vascularized and/or lack mitochondria, formation of lactate is the major fate of pyruvate as seen in lens, cornea of the eye, kidney medulla, testes, leukocytes and red blood cells.
73.

Mother   of   a   2   year    old   child consulted   a  stomatologist.  In  the   period of pregnancy  she was irregularly  taking antibiotics  for an infectious disease. Examination of  the  child  revealed   incisor destruction, yellow enamel, brown rim around the  dental  cervix. What  drug  has apparent teratogenic effect?

Explanation

Doxycycline is a tetracycline, broad spectrum antibiotic. Bacteriostatic; bind to 30s subunit of ribosome. All other options are not antibiotics.

74.

A   59  year   old   patient  is  a  plant manager. After  the  tax  inspection of his plant he felt intense pain behind his breastbone  irradiating  to  his  left  arm. 15 minutes  later his condition came to normal.  Which  of the  possible  mechanisms of stenocardia development is the leading in this case?

 

Explanation

With no pathological condition recorded in the patient. The task of inspection can increase catecholamine concentration in blood due to activation of the sympathoadrenal system. Increased concentration of catecholamines (epinephrine, norepinephrine) can lead to vasospastic, prinzmetal, variant or rest-angina, due to coronary artery spasm. Symptoms are caused by decreased blood flow to the heart muscle from the spasm of the coronary artery.

75.

A patient recovered from Sonne dysentery and was once more infected with the same causative  agent. What is such infection form called?

Explanation

76.

A  56  year   old  patient  came   to  a hospital with complaints about general weakness, tongue pain and burning, sensation  of limb  numbness. In  the  past he underwent resection of forestomach. In blood: Hb- 80 g/l; erythrocytes - 2, 0 · 1012/l; colour  index - 1,2, leukocytes  - 3, 5 · 109/l. What anemia  type is it?

Explanation

Resection of forestomach: Vitamin B12 deficiency, megalocytic, macrocytic anemia.

Resection of pyloric part of stomach: iron deficiency, microcytic anemia.

    Vitamin B12 caused subacute combined degeneration – demyelination of dorsal columns, lateral corticospinal tracts and spinocerebellar tracts of spinal cord which produces paresthesia (sensation of limb numbness) and sensory ataxia. Absorption of Vitamin B12 takes place in the stomach, when it has the castle’s intrinsic factor produced by parietal cells in the stomach. Resection of the forestomach can reduce the intrinsic factor produced, thereby reducing Vit. B12 absorption → Vit. B12 deficiency. The tongue is shiny, glazed and “beefy” (atrophic glossitis).

77.

A   35  year   old   patient  applied   to a  doctor   with   complaints   about   having  intense   rhinitis  and  loss  of  sense  of smell for a week. Objectively:  nasal cavity  contains   a  lot  of  mucus  that  covers mucous  membrane and  blocks  olfactory receptors. In what part of nasal cavity are these receptors situated?

 

Explanation

     Olfactory receptors are situated in olfactory mucus membrane, which is the modified mucus membrane that lines upper part of nostril (superior nasal concha). Olfactory receptor cell is a bipolar neuron. Dendrite of this neuron is short and it has an expanded end called olfactory rod. From the olfactory rod, about 10-12 cilia arise. These cilia project to the surface of olfactory mucus membrane in the upper part of nostril. Mucus secreted by Bowman glands continuously lines the olfactory mucosa.

78.

A  17 year  old  boy  fell  seriously  ill, the  body temperature rose up to 38, 5oC , there  appeared cough,  rhinitis,  lacrimation,  nasal  discharges.  What  inflammation is it?

Explanation

Catarrhal inflammation is one of the morphologic patterns in acute inflammation. In this type, a surface inflammation is associated with greatly increased secretion of clear mucus (nasal discharges). Rhinitis is inflammation of mucous lining of the nose.

79.

A patient with disturbed cerebral circulation   has  problems with  deglutition. What part of brain was damaged?

Explanation

Brainstem is the part of the brain formed by medulla oblongata, pons and midbrain. It contains many centers for regulation of vital functions in the body. Medulla oblongata has many important centers which control vital functions like the respiratory centers; vasomotor center; deglutition center; vomiting center; superior and inferior salivatory nuclei; cranial nerve nuclei; vestibular nuclei. Deglutition center regulates the pharyngeal and esophageal stages of deglutition. Disturbed cerebral circulation can disrupt proper functioning of these centers.

80.

Vitamin  B1    deficiency  results  in  disturbance  of  oxidative   decarboxylation of  α-ketoglutaric  acid.  This  will disturb synthesis of the following coenzyme:

Explanation

        Thiamine (vitamin B1): thiamine pyrophosphate (TPP) is the biologically active form of the vitamin, formed by the transfer of a pyrophosphate group from ATP to thiamine. Biological role of TPP: it is a component of pyruvate dehydrogenase and α-ketoglutarate dehydrogenase complexes catalyzing the reactions of oxidative decarboxylation of pyruvate and α-ketoglutarate (kreb’s cycle) i.e. it promotes energy formation from carbohydrates and lipids. It’s also a component of transketolase (pentose phosphate pathway of glucose oxidation) essential for fats and nucleic acids synthesis.

Pyruvate to acetyl CoA + CO2 reaction

            If pyruvate dehydrogenase cannot function properly due to vitamin B1 deficiency, then pyruvate will be accumulated in blood because it can’t be broken down.

81.

A chemical burn caused esophagus stenosis.  Difficulty of ingestion  led to the abrupt loss of weight. In blood: 3, 0 · 1012/l, Hb - 106 g/l, crude  protein - 57 g/l. What type of starvation is it?

Explanation

Starvation is a stage that appears when the organism doesn’t receive food at all or receives them in insufficient quantity or when the organism doesn’t assimilate it due to a pathology. Starvation can be complete or incomplete (quantitative and qualitative). Complete starvation can be without water (absolute starvation). Incomplete starvation occurs more often than complete starvation. Quantitative incomplete starvation appears when the organism doesn’t chronically receive the necessary quantity of energy with food for energy expenditures (i.e. the organism eats, but it is not enough in quantity). Esophageal stenosis (narrowing of esophagus) will greatly reduce the amount of food taken by the patient thereby leading to incomplete starvation. It is usually marked by a decrease in body mass (weight).

82.

A   30   year   old   woman   has   face edemata. Examination revealed  proteinuria (5,87 g/l), hypoproteinemia, dysproteinemia, hyperlipidemia. What condition is  the  set  of  these  symptoms typical for?

Explanation

Nephrotic syndrome is characterized by massive proteinuria (>3.5g/day) with hyperlipidemia, hypoalbuminemia (hypo- and dysproteinemia), edema etc. It may be primary (direct podocyte damage) or secondary (podocyte damage from systemic process e.g. diabetes). For example – focal segmental glomerulosclerosis, minimal change disease, membranous nephropathy, amyloidosis, diabetic glomerulonephropathy.

83.

Skin  of a man  who  died  from  cardiac insufficiency  has  an  eruption in form of   spots   and   specks.   There    are   also bedsores  in the  area  of sacrum  and  spinous vertebral processes.  Microscopical examination of CNS, skin, adrenal glands revealed  in the vessels of microcirculatory bed and in small arteries destructive-proliferative endothrombovasculitis with Popov’s granulomas; interstitial myocarditis. What  diagnosis  corresponds with the described picture?

Explanation

Rickettsiae are obligate intracellular parasites because they are unable to produce sufficient energy to replicate extracellularly. They are the agents of typhus, spotted fevers and Q fever. The typical lesion caused by the rickettsiae is a vasculitis (endothrombovasculitis), particularly in the endothelial lining of the vessel wall where the organism is found. Damage to the vessels of the skin results in the characteristic rash and in edema, and hemorrhage caused by increased capillary permeability. Rickettsia rickettsii causes Rocky Mountain spotted fever. In rickettsioses, especially in louseborne typhus, which is caused by Rickettsia prowazekii and in Rocky Mountain spotted fever, which is caused by R. rickettsii, the formation of typhus nodules in the small vessels results in a proliferative destructive thrombovasculitis with formation of perivascular granulomas (popov’s granulomas) consisting of macrophages, lymphocytes and plasma cells.
84.

Autopsy  of  a  man  who  died  from the sepsis in his femoral bone revealed phlegmonous inflammation that  affected the  marrow,   haversian canals  and  periosteum.  Under the  periosteum there  are multiple  abscesses,  adjoining  soft  tissues of thigh  also have  signs of phlegmonous inflammation. What  pathological process was described?

Explanation

Osteomyelitis is the inflammation of bone caused by pyogenic organisms. Acute osteomyelitis is an infection of the bone of less than two weeks’ duration, which typically spreads hematogenously (acute hematogenous osteomyelitis). A delay in diagnosis may cause growth disturbance, deformity or even death. It typically affects the long bones (femur and tibia are the most commonly affected). Pathogenesis: preexisting focus/exogenous infection → infective embolus enters nutrient artery → trapped in a vessel in metaphysis and blocks the vessel → active hyperemia and polymorphonuclear cell exudates → hyperemia and immobilization causes decalcification proteolytic enzymes destroy bacteria and medullary elements → the debris increase and intramedullary pressure increases → follows paths of least resistance → passes through haversian canal and Volkmann canal → local cortical necrosis → enter subperiosteal space → strips periosteum → perforation of periosteum/reach joint by piercing capsule → enters soft tissue and may drain out.

85.

A patient has a decreased vasopressin synthesis  that  causes  polyuria   and  as  a result  of it evident  organism  dehydration.  What  is the  mechanism   of  polyuria development?

Explanation

        Antidiuretic hormone (vasopressin) is secreted in response to decrease blood volume and increase plasma osmolarity. It binds to receptors on principal cells of collecting ductules causing increase number of aquaporins and increase water reabsorption which leads to decreased diuresis.

↓vasopressin → ↓H2O reabsorption → polyuria

86. A  patient with  nephrotic  syndrome has  massive  edemata of his face  and  limbs. What is the leading pathogenetic mechanism  of edemata development?

Explanation

Nephrotic syndrome is characterized by massive proteinuria (>3.5g/day) with hyperlipidemia, hypoalbuminemia (hypo- and dysproteinemia), edema etc. It may be primary (direct podocyte damage) or secondary (podocyte damage from systemic process e.g. diabetes). For example – focal segmental glomerulosclerosis, minimal change disease, membranous nephropathy, amyloidosis, diabetic glomerulonephropathy.

87.

Examination of a 55 year  old woman revealed   under   the   skin  of  submandibular area a movable slowly growing pasty formation with distinct borders 1,0x0,7 cm large.  Histological  examination revealed lipocytes that form segments of different forms and sizes separated from each other by  thin  layers  of  connective tissue  with vessels. What is the most probable diagnosis?

Explanation

Lipoma is a benign tumor of fatty tissue. It is the most frequent soft tissue tumor, arising in subcutaneous regions at any site but most commonly on the back, shoulder and neck. It may develop in every site where there is fat tissue. Lipomas are encapsulated, usually small yellow node with distinct (clear) borders.

Liposarcoma and fibrosarcoma are malignant with no distinct borders. Angioma is tumor of vessels; Fibroma and Fibrosarcoma are tumors of connective tissue.

88.

An unconscious  young man with signs of  morphine poisoning  entered admission  office. His  respiration is shallow  and infrequent which  is caused  by inhibition of respiratory centre.  What  type of respiratory failure is it?

Explanation

Morphine is the major analgesic drug contained in crude opium. Opoids exert their major effects by interacting with opoid receptors in the CNS and in other anatomic structures, such as the GIT and the urinary bladder. Severe respiratory depression can occur and result in death from acute opoid poisoning. A serious effect of the drug is stoppage of respiratory exchange in patients with emphysema or Cor pulmonale. In this case, there is dysregulation of the respiratory center in the CNS.

89. A   patient  with   cholelithiasis    fell ill  with  mechanic   jaundice.   Examination  revealed   that   the  stone   was  in  the common   bile  duct.  What   bile-excreting ducts make up the obturated duct?

Explanation

The common bile duct (ductus choledochus) forms in the porta hepatica as the result of merging of the common hepatic duct (ductus hepaticus communis) and the cystic duct (ductus cysticus). It is approximately 5-8cm long.
90.

A patient is 44 years old. Laboratory examination of  his  blood  revealed   that content of proteins  in plasma  was 40 g/l. What   influence   will  be  exerted on  the transcapillary water exchange?

Explanation

         Reference interval (adults): 60 – 80g/L

40g/L is low i.e. a low oncotic pressure.

Glomerular filtration rate (GFR) is inversely proportional to oncotic pressure/colloid osmotic pressure, which is exerted by plasma proteins in the glomerular capillary blood. When oncotic pressure is low, GFR increases. Reabsorption is decreased, because the low oncotic pressure is not sufficient to reabsorb water and other substances (cannot create enough pressure to reabsorb water and other substances).

91.

The cerebrospinal fluid is being examined for the purpose of differential meningitis diagnostics.  At what site is the lumbar puncture safe?

Explanation

Ligamentum flava resides between the vertebral arches and consist of yellow elastic tissue. The clefts between the vertebral arches are covered by the ligamentum flava, which is the widest in the lumbar region. Therefore, these regions are used for the punctures of the vertebral canal to access the subarachnoid space. This procedure is actually performed on the L2 and L3 as well between the L3 and L4. Moreover, the puncture is also performed between the occipital bone and the first cervical vertebra piercing the atlanto-occipital membrane. In the thoracic region, the spinous processes overlap each other like a tile covering the arches of the lower vertebrae.

92.

An isolated  cell of human  heart automatically generates excitement impulses  with frequency of 60 times  per minute. This cell was taken from the following heart  structure:

Explanation

The sinoatrial (SA) node is a small strip of modified cardiac muscle, situated in the superior part of lateral wall of right atrium, just below the opening of superior vena cava. The fibers of this node do not have contractile elements. SA node is called the pacemaker because the rate of production of impulse (rhythmicity) is higher in SA node than in other parts. The rate is 60-100/min and higher during tachycardia when stimulated by sympathetic effects. Atrioventricular (AV) node is 40-60/min.

93.

Examination of a 60 year old patient revealed  hyperglycemia and glucosuria. A doctor  administered him a medication for internal use. What medication is it?

Explanation

Sulfonylurea are insulin secretagogues because they promote insulin release from the β-cells of the pancreas. The primary drugs used today are the 2nd generation drugs – glibenclamide, glipizide, glimepiride etc. These agents are useful in the treatment of patients who have type 2 diabetes mellitus and cannot be managed by diet alone. Metformin is the only currently available biguanide; its classed as an insulin sensitizer. Acarbose – α glucosidase inhibitor. Butamide (tolbutamide) is also a sulfonylurea but is older and glibenclamide is more potent and used much more commonly. Actrapid (soluble insulin) is fast acting but not a sulfonylurea.

94.

As  a result  of posttranslative modifications some proteins taking part in blood coagulation, particularly prothrombin, become    capable    of   calcium    binding. The  following  vitamin  takes  part  in this process:

Explanation

Vitamin K-dependent factors include factor II, VII, IX and X; protein C and S. vitamin K is activated in the liver by epoxide reductase. Activated vitamin K ɣ-carboxylates each of the vitamin K-dependent factors. Carboxylated factors are now able to bind calcium, which are essential for interaction between the coagulation factors and platelet membranes.

Factor II – Prothrombin

Factor VII – Stable factor

Factor IX – Christmas factor

Factor X – Stuart-Prower factor

These factors undergo vitamin K-dependent post-translational modification, whereby a number of their glutamic acid residues are carboxylated to form ɣ-carboxyglutamic acid residues. The ɣ-carboxyglutamyl residues bind calcium ions which are essential for interaction between the coagulation factors and platelet membranes.

Warfarin inhibits the enzyme vitamin K epoxide reductase. Neonates lack enteric bacteria which produce vitamin K.

95.

Examination of a man who had been working  hard  under  higher  temperature of the environment revealed abnormal quantity of blood  plasma  proteins. What phenomenon is the case?

Explanation

Liver is the primary organ of protein synthesis. In a case of dysfunction of protein synthesis, hypoproteinemia occurs. We have absolute and relative hypoproteinemia. Absolute is when there is disturbed synthesis (absolute hypoproteinemia) and in cases of increased synthesis (absolute hyperproteinemia); while relative does not have to do with synthesis but other pathologies e.g. renal lesions, in which case, there is loss of protein with urine (relative hypoproteinemia); also vomiting, diarrhea, profuse sweating can produce relative hyperproteinemia.

It is Relative because it does not have to do with synthesis. The man must have lost a lot of fluid through sweat, thereby decreasing the fluid content of the body and creating a relative hyperproteinemia. Decrease blood volume (liquid component alone) → ↑concentration of blood.

96.

A patient ill with thrombophlebitis of his lower limbs had chest pain, blood spitting, progressing  respiratory insufficiency that  led to his death.  Autopsy diagnosed multiple lung infarctions. What is the most probable cause of their development?

 

Explanation

Pulmonary embolism is the most common and fatal form of venous thromboembolism in which there is occlusion of pulmonary arterial tree by thromboemboli. Pulmonary emboli are more common in hospitalized or bedridden patients. The majority of emboli arise from the deep veins of the lower extremities; most of the fatal ones arise from the ileofemoral veins. Consequences of thromboembolism include pulmonary infarction in which pyramidal segments (or ‘triangular’ segments) of hemorrhagic infarction are seen at the periphery of the lung. Detachment of thrombi from the thrombophlebitis site in the lower limbs produces a thromboembolus that flows through venous drainage into the large veins draining into the right side of the heart and then to the pulmonary circulation where they embolize.

97.

A woman suffering from osteochondrosis felt acute pain in her humeral articulation that  became  stronger when she abducted her  shoulder. These  symptoms  might  be caused by damage of the following nerve:

Explanation

Deltoid muscle …

 Origin: lateral third of clavicle, acromion of scapula, spine of scapula.

Insertion: Deltoid tuberousity of humerus.

Innervation: Axillary nerve ( a terminal branch of branchial plexus receiving fibres from C5 & C6 ventral rami.

Action:

Anterior part- flexion & medial (internal) rotation of the humerus at glenohumeral  joint.

Middle part- abduction of the humerus at the glenohumeral joint.

Posterior part – extension & lateral (external) rotation of the humerus at the glenohumeral joint.

 
98.

A patient has a deep cut wound on the posterior surface of his shoulder in its middle third. What muscle might be injured?

Explanation

Triceps brachii is the chief extensor of the forearm at the humeroulnar joint. Has a long, lateral, and medial head. Inserts on olecranon of ulna bone.                                                                                 

99. A patient has acne on his face. Microscopic  examination of scrapings from the affected  areas  revealed  living porrect vermiform   arthropoda 0,2-0,5 mm  large with four pairs of short  extremities in the front part of their bodies. What is the laboratory diagnosis?

Explanation

          Demodex folliculorum and Demodex brevis: they are species of face mite. When large numbers of D. folliculorum are found in humans, the infestation is known as Demodicosis. D. folliculorum is semi-transparent elongated organism consisting of a head, neck, body and tail. As an adult, it can measure 0.1-0.4mm in length and possess 4 pairs of short legs near its head and neck region. In the course of time, because of chronic progradient course, pathologic process results in formation of inflammatory nodes, infiltrates and persistent vascular dilatation. Hyperemia of facial skin and eruptions are also seen.

100.

A  28 year  old  man  had  a  gunshot wound    of   shin   that    resulted   in   an ulcer  from  the  side  of  the  injury.  What is the main factor of neurodystrophy pathogenesis in this case?

Explanation

The concept of trophic function of the nervous system arose early in the 19th century owing to the attempts of clinicians and physiologists to account for the pathological changes occurring in tissues (ulcers, necrosis and atrophy) upon injury to the nervous system (trauma to peripheral nerve). These changes, known as neurogenic dystrophies take place when neuroregulation of metabolism in tissues is impaired by injury or chronic irritation of nerve trunks or by injury to the central nervous system, in particular, to the hypothalamus. In studies on the trophic function of the nervous system made during the 1920s, I. P. Pavlov  maintained that every organ is controlled by the nerves in 3 ways: Functional control arouses or arrests the activity of organs; Vascular control regulates the supply of nutrients by the blood and Trophic control regulates the utilization of nutrients by organs.

101.

A 45 year old man consulted  a doctor about a plaque-like formation on his neck. Histological   examination  of  a  skin  bioptate  revealed   clusters   of  round   and oval tumour cells with a narrow  border of basophilic  cytoplasm  resembling of cells of basal epidermal layer. What  tumour is it?

Explanation

Skin cancer (epithelial tumors) is divided into 4 groups:

·        Intraepidermal cancer (cancer in situ): Bowen’s disease, erythroplasia of Queyrat.

·        Basal cell carcinoma (BCC, basalioma)

·        Epidermoid cancer

·        Skin appendage cell carcinoma: Paget’s disease

Basal cell carcinoma (basalioma) is malignant; the tumor is characterized by slow growth (sometimes for years), locally destructive invasive growth without metastases and frequent recurrences. The tumor is of polymorphous structure derived from the epidermis and skin appendages. The most prevalent location is the face, neck and scalp. Gradually increasing in size, they can form a large plaque or node with ulcerative depression in the center and elevated borders. It has the appearance of a flat well-defined reddish, round or oval-shaped spot or plaque.

102.

A patient ill with essential hypertension  was  recommended a  drug that prevents thrombosis. It is to be taken parenterally. What drug is it?

Explanation

Heparin is a natural anticoagulant produced in mast cells and basophils. It is an injectable, rapidly acting anticoagulant that is often used acutely to interfere with the formation of thrombi. Heparin is used in the prevention of venous thrombosis and the treatment of a variety of thrombotic diseases such as pulmonary embolism and acute myocardial infarction. Heparin binds to antithrombin III, with the subsequent rapid inactivation of coagulation factors. Antithrombin III inhibits serine proteases, including several of the clotting factors, most importantly, thrombin (factor IIa) and Factor Xa (a-active).

Protamine sulphate antagonizes the anticoagulant effects of heparin. The positively charged protamine interacts with the negatively charged heparin forming a stable complex without anticoagulant activity.

103.

Packed   cell  volume  of  a  man  was 40% before  the trauma. What packed  cell volume  will be  observed 24 hours  after blood loss of 750 ml?

Explanation

Blood loss during trauma results in loss of all components of blood (cells and liquid component of blood). Therefore, the packed cell volume (PCV) or Hematocrit value will decrease after a blood loss. Erythropoiesis cannot replace all cells lost in 24hours. Therefore, the Hematocrit or PCV should not be up to 40% 24 hours after the bloodloss of about 750ml.

104. Two hours after an exam a student had   a   blood   count   done   and   it   was revealed   that   he   had   leukocytosis   without significant leukogram modifications. What  is the most probable mechanism  of leukocytosis  development?

Explanation

The exam is a form of psychological stress causing the release of Stress hormones (catecholamines, cortisol). Stress-induced changes in blood leukocyte distribution may represent an adaptive response. This represents a redistribution of leukocytes from the blood to other organs such as the skin, draining sentinel lymph nodes and other compartments. Such leukocyte redistribution may enhance immune function in compartments to which immune cells traffic during stress.

105.

Golgi  complex   exports   substances from    a   cell   due    to    the    fusion    of the membrane saccule with the cell membrane.  The   saccule   contents  flows out. What process is it?

Explanation

 Exocytosis is the process by which a vesicle moves from the cytoplasm to the plasma membrane, where it discharges its contents to the extracellular space. A variety of molecules produced by the cell for export are initially delivered from the site of their formation to the golgi apparatus. The next step involves sorting and packaging the secretory product into transport vesicles that are destined to fuse with the plasma membrane in a process known as exocytosis.

106.

A boy found a spider with the following morphological characteristics: it is 2 cm long,  has  roundish black  abdomen with two  rows of red  spots  on its dorsal  side; four pairs of jointed limbs are covered with small black hairs. What arthropod is it?

Explanation

Karakurt spider: the word kara meaning “black” and kurt meaning “worm” comes from the Turkic languages. It is black in colour and is identified by the thirteen (13) spots which are found on its dorsal abdomen. These spots are usually red in colour, but may also be yellow, or orange. The female has a body length of 1 – 2cm (10-20mm), while the male is smaller and reaches 0.5 – 0.8cm (5-8mm) at best. Only the female spider’s bite is dangerous (either for humans or cattle) as the male cannot penetrate the relatively thick epidermis.

107.

A  3 year  old  child  with  symptoms of   stomatitis,    gingivitis   and    dermatitis   of   open   skin   areas   was   delivered to a hospital. Examination revealed inherited  disturbance  of  neutral  amino acid transporting in the bowels. These symptoms  were  caused  by the  deficiency of the following vitamin:

 

Explanation

108. A  patient with  suspected   diagnosis \\\"progressing muscular  dystrophy\\\" got his urine tested. What compound will confirm this diagnosis if found in urine?

Explanation

Creatine kinase/Creatine phosphokinase is an enzyme present in striated muscles, used to generate energy. When its serum level is elevated it is often an indication of muscle damage caused by injury, muscular dystrophy or cardiac problems. Most forms of muscular dystrophy are associated with decreased creatinine excretion. Creatinine is a break down product of creatine. Normal constituent of urine but can be elevated in muscular dystrophy. Since creatine is not a normal constituent of urine, it is more specific in this case.

109.

A patient being treated for viral hepatitis type B got symptoms  of hepatic insufficiency. What blood changes indicative of protein metabolism disorder will be observed in this case?

Explanation

Liver is the primary organ of protein synthesis. In a case of dysfunction of protein synthesis, hypoproteinemia occurs. We have absolute and relative hypoproteinemia. Absolute is when there is disturbed synthesis (absolute hypoproteinemia) and in cases of increased synthesis (absolute hyperproteinemia); while relative does not have to do with synthesis but other pathologies e.g. renal lesions, in which case, there is loss of protein with urine (relative hypoproteinemia); also vomiting, diarrhea, profuse sweating can produce relative hyperproteinemia.

Albumin is a major component of plasma protein ( proteinemia - albuminemia)

110. A  pregnant woman  had  her  blood group  identified.  Reaction of erythrocyte agglutination with standard serums of 0αβ (I),  Bα  (III)   groups  didn’t  proceed with standard serum  of Aβ  (II)  group.  The blood group under  examination is:d

Explanation

Blood group O(I): no antigens, therefore no agglutination.

Blood group A(II): A antigen, agglutinate with blood group B(III) and O(I).

Blood group B(III): B antigen, agglutinate with blood group A(II) and O(I).

Blood group AB(IV): A and B antigen, agglutinate with all blood groups. No antibody.

Blood group name is determined by the antigen present on RBC, but the patient has an opposite antibody. So whenever, the antibody corresponds to the antigen, there is agglutination. Since O does not have any antigen, no agglutination can occur.

111. A sensitive  neural  ganglion  consists of roundish neurocytes with one  extension that  divides into axon and dendrite at some distance  from the perikaryon. What are these cells called?

Explanation

        Neurons are classified on the basis of the number of processes extending from the cell body:

·        Pseudounipolar: neurons have one process (extension), which divides close to the cell body into two long processes (axon and dendrite). The vast majority are located in the dorsal root ganglia and cranial nerve ganglia.

·        Unipolar neurons have only one pole. From a single pole, both axon and dendrite arise. This type of nerve cells is present only in embryonic stage in human beings.

·        Bipolar neurons have two poles – one axon and one dendrite.

·        Multipolar neurons have many poles – one axon and two or more dendrites.

112. A doctor examined a child and revealed symptoms of rachitis. Development of this disease  was caused by deficiency of the following compound:

Explanation

Rickets results from insufficiency of vitamin D (calcitriol). Calcitriol is the active form of vitamin D. calcitriol production is dependent on the kidney’s 1-α-hydroxylase which converts 25-OHD3 to 1,25-(OH)2D3 (calcitriol). So in cases of renal lesion, there is lack of 1-α-hydroxylase which leads to a deficiency of calcitriol or impaired synthesis of calcitriol.

113.

An embryo displays disturbed process of dorsal mesoderm segmentation and  somite  formation. What  part  of skin will have developmental abnormalities?

Explanation

The embryonic period or period of organogenesis occurs from the third – eighth weeks of development and is the time when each of the 3 germ layers: ectoderm, mesoderm and endoderm, give rise to a number of specific tissues and organs.

Mesoderm: paraxial, intermediate and lateral mesoderm. By the beginning of the 3rd week, paraxial mesoderm begins to be organized into segments. These segments, known as somitomeres, which further organize into somites (42-44 pairs). Somites: sclerotome (tendon, cartilage and bone – vertebrae and rib cage); Myotome (muscles); Dermatome (dermis). From their initial location within the somite, the sclerotome cells migrate medially towards the notochord.
114.

It  was  found  out  that  some compounds, for instance  fungi toxins and some  antibiotics   can  inhibit   activity  of RNA-polymerase.  What  process  will be disturbed in a cell in case of inhibition  of this enzyme?

 

Explanation

     The process of transcription of a typical gene can be divided into 3 phases: initiation, elongation and termination. The process of transcription begins with the binding of the RNA polymerase holoenzyme to a region of the DNA known as the promoter. RNA polymerase is a multisubunit enzyme that recognizes a nucleotide sequence (the promoter region) at the beginning of a length of DNA that is to be transcribed. It makes a complementary RNA copy of the DNA template strand and then recognizes the end of the DNA sequence to be transcribed (the termination region).

115.

A 7 year old child is ill with bronchitis. It  is necessary to administer him an antibacterial drug. What   drug of fluoroquinolone group is CONTRA- INDICATED at this age?

Explanation

Fluoroquinolones: ciprofloxacin, norfloxacin, levofloxacin, ofloxacin, moxifloxacin. They are all antibiotics. They inhibit prokaryotic enzymes topoisomerase II (DNA gyrase) and topoisomerase IV. They are bactericidal and must not be taken with antacids. It is contraindicated in pregnant women, nursing mothers and children

Fluoroquinolones hurt attachments to your Bones.

116.

A culture  of monkey  cells (Vero) and a group of mouse  sucklings were infected with an inoculum  taken  from a child with provisional  diagnosis \\\"enterovirus infection\\\".  There  was  no  cytopathic effect  on the cell culture  but mouse  sucklings died. What enteric viruses might have caused disease of this child?

 

Explanation

     Picornaviruses are small nonenveloped viruses composed of an icosahedral nucleocapsid and a single-stranded RNA genome (ssRNA). Picornavirus family ncludes two groups of medical importance: the enteroviruses and rhinoviruses. Among the major enteroviruses are the poliovirus, coxsackieviruses, echoviruses and hepatitis A virus.

Coxsackieviruses are named for the town of Coxsackie, NY, where they were first isolated. Coxsackievirus cause a variety of diseases. Coxsackieviruses are divided into group A and group B viruses based on early observations of their pathogenicity in newborn mice (mouse sucklings). Group A causes flaccid paralysis while group B causes spastic paralysis. Coxsackie A virus causes paralysis and death of the mice; coxsackie B causes less severe infection in the mice.

117.

A patient died from cardiopulmonary decompensation. Histological  examination   revealed    diffused    pulmonary   lesion together with interstitial edema, infiltration of tissue by limphocytes, macrophages, plasmocytes;  pulmonary fibrosis,  panacinar  emphysema. What  disease  corresponds with the  described  picture?

Explanation

        Fibrosing alveolitis is also known as interstitial pneumonia. It is characterized by progressive scarring of both lungs. The scarring (fibrosis) involves the supporting framework (interstitium) of the lung. There are patchy or lobar areas of congestion without the consolidation of bacterial pneumonias. A predominance of interstitial pneumonitis with widened, edematous alveolar walls containing a mononuclear inflammatory cell infiltrates. Interstitial pneumonia is the most common form of interstitial fibrosis.

118. Vagus branches that innervate heart are being stimulated in course of an experiment.  As  a  result  of  it  the  excitement conduction from atria  to the ventricles   was  brought  to  a  stop.  It  is caused  by electrophysical changes  in the following structures:

Explanation

Atrioventricular (AV) node serves as the gateway for conduction of impulses from the atria to the ventricle. Stimulation of the parasympathetic nerves to the heart (vagus nerve) causes the neurotransmitter acetylcholine to be released at the vagal endings. This neurotransmitter has two major effects on the heart. First, it decreases the rate of rhythm of the sinus node, and second, it decreases the excitability of the AV junctional fibers between the atrial musculature and the AV node, thereby slowing transmission of the cardiac impulse into the ventricles. In the AV node, a state of hyperpolarization caused by vagal stimulation makes it difficult for the small atrial fibers entering the node to generate enough electricity to excite the nodal fibers. A moderate decrease simply delays conduction of the impulse, but a large decrease blocks conduction entirely.

119.

If a man  has  an  attack  of bronchiospasm it is necessary to reduce  the effect of vagus  on  smooth  muscles  of bronchi. What  membrane cytoreceptors should  be blocked for this purpose?

Explanation

Muscarinic receptors belong to the class of G protein-coupled receptors. These receptors, in addition to binding Acetylcholine, it also recognizes muscarine – an alkaloid that is present in certain poisonous mushrooms. Muscarinic effects include miosis, spasm of accommodation, increasing of lacrimal, salivary, gastrointestinal, bronchial and sweat glands secretion, increase of smooth muscle tonus of internal organs, bradycardia, decreased arterial pressure.

M-cholinoreceptors increase muscle tonus of internal organs. Therefore blocking this receptor will relieve the bronchospasm.

120.

A patient of surgical department complains  about  pain  in the  small of her back  and  in the  lower  part  of her  belly; painful  and  frequent  urination. Bacteriological examination of urine revealed gram-negative oxidase-positive rod-like bacteria forming  greenish  mucoid  colonies  with  specific  smell.  What   causative agent can it be?

Explanation

Pseudomonas produces blue green pigment; Yellow-green and red-brown pigments. 
121.

There     are     several     groups     of molecular mechanisms  playing important part   in  pathogenesis  of  insult   to  cells which contributes to the pathology development. What processes are stimulated  by  proteinic   damage   mechanisms?

Explanation

          Protein mechanism is one of the molecular mechanisms of cell injury. The protein mechanism of cell damage includes:

·        Inhibition of enzymes (reversible and irreversible)

·        Denaturation – violation of native structure of proteins

·        Proteolysis – that is carried out under the action of lysosomal enzymes.

122. A  child  was born  with  cleft  palate. Examination revealed  aorta  defects  and reduced  number    of   T-lymphocytes   in blood.  What  immunodeficient syndrome is it?

Explanation

DiGeorge’s syndrome results from a congenital malformation affecting the 3rd and 4th pharyngeal pouches. These structures give rise to the thymus, parathyroid glands and portions of lips, ears and the aortic arch. It is the most common disorder of cell mediated immunity. The thymus is usually rudimentary and T cells are deficient or absent in the circulation. Most of these infants have additional developmental defects affecting the face, ears, lips heart and great vessels. All of these disorders are part of a spectrum called 22q deletion syndrome because they result from a deletion on the long arm of chromosome 22 (22q11). It represents the most severe example of a collection of disorders that also includes velocardiofacial syndrome (VCFS) and conotruncal anomalies face syndrome.

123.

A 16 year old boy after an illness has diminished  function  of protein synthesis in liver as a result of vitamin K deficiency. It will cause disturbance of:

Explanation

Vitamin K is essential for the formation of various clotting factors in the liver, namely clotting factor II, VII, IX and X; Protein C and S.

Factor II – Prothrombin

Factor VII – Stable factor

Factor IX – Christmas factor

Factor X – Stuart-Prower factor

These factors undergo vitamin K-dependent post-translational modification, whereby a number of their glutamic acid residues are carboxylated to form ɣ-carboxyglutamic acid residues. The ɣ-carboxyglutamyl residues bind calcium ions which are essential for interaction between the coagulation factors and platelet membranes. This oral anticoagulants block epoxide reductase and creation of active form of vitamin K resulting in disturbances in prothrombin and proconvertin synthesis in liver.

Deficiency of vitamin K → ↓clotting factors → prolongation of coagulation time → hemorrhages.

124.

Life  cycle  of  a  cell  includes   the process  of DNA  autoreduplication. As a result  of it monochromatid chromosomes turn  into  bichromatid ones.  What  period of  cell  cycle  does  this  phenomenon fall into?

Explanation

Somatic cell division is a cyclic process divided into two phases: mitosis (M phase) and interphase. Three other phases, Gap 1 (G1); synthesis phase (S) and Gap 2 (G2) further subdivide interphase. M phase is followed by G1.

M phase: karyokinesis – division of the nucleus into two daughter nuclei.

                  Cytokinesis – division of the cell into two daughter cells.

G1: a period in which no DNA synthesis occurs.

Period of cell growth S or DNA synthesis phase follows G1 phase. The DNA of the cell is doubled. The S phase is followed by a period in which no DNA synthesis occurs, a second gap or G2 phase. A cell that leaves the cycle in G1 phase to begin “terminal” differentiation enters the G0 phase, (“O” stands for “outside” the cycle).

125.

A driver who got a trauma in a road accident  and  is shocked  has reduction of daily urinary output down to 300 ml. What is the main pathogenetic factor of such diuresis change?

Explanation

126.

Systemic arterial pressure of an adult dropped  from  120/70  to  90/50  mm  Hg that led to reflectory vasoconstriction. The vasoconstriction  will be  maximal  in  the following organ:

Explanation

     The drop in arterial pressure (AP) will cause a reflex tachycardia, the heart increases its contractility and cardiac output. Vasoconstriction will be minimal in the heart, because the heart needs to pump more i.e. do more work. Therefore, it needs more blood supply. But vasoconstriction will be maximal in the GIT (bowels), muscles, skin due to centralization of blood flow (i.e. redistribution of blood to important organs like heart, brain)

127.

Examination   of   a   12   year    old boy with developmental lag revealed achondroplasia: disproportional constitution  with  evident  shortening of  upper and lower limbs as a result of growth disorder of epiphyseal cartilages  of long tubal bones. This disease is:

Explanation

Autosomal dominant: often due to defects in structural genes. Many generations, both male and female are affected. Found in every generation; no generation is left out (skipped). Parent – child in every generation.

Autosomal recessive is usually seen in some generations (other generations are skipped).

It is sex linked, if only males or only females(X-linked) are affected.

128.

A patient was diagnosed with autoimmune  hemolytic cytotoxic anemia.  What substances  are antigens  in II type allergic reactions?

Explanation

129.

While performing an operation in the area  of axillary  crease  a surgeon  has  to define  an  arterial vessel  surrounded by fascicles of brachial plexus. What artery is it?

Explanation

130.

A patient takes digoxin for treatment of   cardiac   insufficiency.   What   diuretic may increase  digoxin  toxicity  due  to the intensified excretion of K +  ions?

 

Explanation

         Hydrochlorothiazide is a thiazide diuretic. Thiazide diuretics induce hypokalemia and hyperuricemia in 70% of patients and hyperglycemia in 10% of patients. Serum potassium levels should be monitored closely in patients who are predisposed to cardiac arrhythmias (particularly individuals with left ventricular hypertrophy, ischemic heart disease or chronic heart failure) and those who are concurrently being treated with both thiazide diuretics and digoxin. In general, decreased serum levels of potassium predispose a patient to digoxin toxicity.

131.

During  starvation muscle proteins break up into free amino acids. These compounds will be the most probably involved into the following process:

Explanation

         Gluconeogenesis is making of glucose from non-carbohydrate sources i.e. fat and proteins. During a prolonged fast, hepatic glycogen stores are depleted and glucose is formed from precursors such as lactate, pyruvate, glycerol (derived from backbone of triacylglycerols) and α-ketoacids (derived from the catabolism of glucogenic amino acids). Amino acids derived from hydrolysis of tissue proteins (e.g. muscle proteins) are the major sources of glucose during a fast. α-ketoacids such as oxaloacetate and α-ketoglutarate are derived from the metabolism of glucogenic amino acids. Amino acids whose catabolism yields pyruvate or one of the intermediates of citric acid cycle (CAC) are termed glucogenic or glycogenic e.g. alanine, arginine, aspartate, histidine etc.

132.

A patient ill with amebiasis  was prescribed  a  certain   drug.   The   use  of alcohol  together with this drug is contra- indicated because  the drug inhibits metabolism of ethyl  alcohol.  What  drug is it?

 

Explanation

Metronidazole forms toxic free radical metabolites in the bacterial cell that damage DNA. It is bactericidal, and an antiprotozoal. It is used to treat Giardia, Entamoeba, Trichomonas, Gardnerella vaginalis, Anaerobes (bacteroides, Clostridium difficile). It can be used with a proton pump inhibitor and clarithromycin for “triple therapy” against Helicobacter pylori.

133.

Surgical removal of a part of stomach resulted in disturbed absorption of vitamin  B12 , it is excreted with  feces.  The patient was diagnosed  with anemia.  What factor  is necessary  for absorption of this vitamin?

Explanation

Resection of forestomach: Vitamin B12 deficiency, megalocytic, macrocytic anemia.

Resection of pyloric part of stomach: iron deficiency, microcytic anemia.

    Vitamin B12 caused subacute combined degeneration – demyelination of dorsal columns, lateral corticospinal tracts and spinocerebellar tracts of spinal cord which produces paresthesia (sensation of limb numbness) and sensory ataxia. Absorption of Vitamin B12 takes place in the stomach, when it has the castle’s intrinsic factor (gastromucoprotein) produced by parietal cells in the stomach. Resection of the forestomach can reduce the intrinsic factor produced, thereby reducing Vit. B12 absorption → Vit. B12 deficiency. The tongue is shiny, glazed and “beefy” (atrophic glossitis).

134.

A   patient  ill  with   chronic   cardiac         insufficiency     was    prescribed    an average  therapeutic dose of digoxin. Two weeks   after   begin   of  its  taking   there appeared symptoms  of drug  intoxication (bradycardia, extrasystole, nausea). Name the phenomenon that  caused  accumulation of the drug in the organism?

 

Explanation

Accumulation is enhancing of the effects in repeated doses that leads to the appearance of toxic effects. It can be:

·        Material accumulation: is accumulation of drug in the manner of bindings with proteins, phospholipids etc.

·        Functional accumulation is accumulation of the drug’s effect in spite of its elimination.

135.

In  course   of  a  preventive  examination   of   a   miner   a   doctor   revealed changes   of  cardiovascular  fitness  which was indicative  of cardiac  insufficiency  at the compensation stage. What is the main proof of cardiac compensation?

Explanation

Enlargement of the muscular wall of the ventricles (myocardial hypertrophy) is an important compensatory mechanism of the heart in cardiac failure. When the heart must work harder, the heart’s wall enlarge and thicken. At first, the thickened heart walls can contract more forcefully. However, the thickened heart walls eventually become stiff, causing or worsening diastolic dysfunction. Myocardial hypertrophy is a long-term compensatory mechanism. Cardiac muscle, like skeletal muscle, responds to an increase in work demands by undergoing hypertrophy. Hypertrophy increases the number of contractile elements in myocardial cells as a means of increasing their contractile performance.

136. Rest  potential of a cell equals  80 mV . At what stage of action potential did the membrane potential equal +30 mV ?

Explanation

At rest, the axon membrane is slightly polarized to about -70mV (or -80mV), meaning the intracellular fluid (ICF) is relatively negative to the extracellular fluid (ECF). An action potential occurs when a portion of the membrane rapidly depolarizes and then repolarizes again to the original resting state. The process is initiated by a threshold level stimulus, such as a nearby change in membrane potential (threshold potential, local potential). At threshold (about -55mV), many Na+ voltage-gated channels open. Na+ ions entering the cell make the membrane potential less negative. More Na+ channels open as a result and a cycle of depolarization develops. When the membrane potential reaches about +30mV (reverse polarization), the timed Na+ channels close due to inactivation and the Na+ influx stops. K+ voltage-gated channels open as Na+ channels close, in a delayed response to the original stimulus.

137.

A  35  year  old  man  got  an  injury that  caused  complete disruption of  spinal  cord  at  the  level of the  first cervical segment.   What  respiration changes  will be observed?

Explanation

138.

In  course  of an  experiment a peripheral section of vagus of an experimental animal is being stimulated. What changes will be observed?

 

Explanation

Vagus nerve gives parasympathetic innervations to the heart. The vagal fibers are distributed mainly to the atria and not much to the ventricles, where the power contraction of the heart occurs. This explains the effect of vagal stimulation mainly to decrease heart rate rather than to decrease greatly the strength of heart contraction.

139.

2  hours   after   a  skeletal   extension was performed to  a 27 year  old  patient with  multiple   traumas (closed  injury  of chest,  closed  fracture of right  thigh)  his condition abruptly became  worse and the patient died from acute  cardiopulmonary decompensation. Histological  examination of pulmonary and cerebral vessels stained with Sudan III revealed  orange  drops occluding the vessel lumen. What complication of polytrauma was developed?

Explanation

Embolism is occlusion of a vessel by material travelling in the circulation.

* Fat/Adipose embolism: obstruction of arterioles and capillaries by fat globules constitutes fat embolism. It may occur following severe fracture trauma to bones, inflammation of bones and soft tissues, fatty liver

* Thromboembolism: a detached thrombus or part of thrombus which may arise in the arterial or venous circulation.

* Gas embolism: two main forms of gas embolism are air embolism and decompression sickness. Air embolism is usually due to accidental pumping of air into the venous circulation during intravenous (IV) injection or transfusion ( bubble – air escaped).

Tissue embolism: fragments of tissue.

140.

In  course  of  severe  respiratory viral  infection   there   appeared  clinical  signs  of  progressing   cardiac   insufficiency that  caused  death  of a patient in the 2nd week  of  disease.  Autopsy revealed   that the  heart   was  sluggish,  with  significant cavity  dilatation. Histological  examination  of myocardium revealed  plethora of microvessels   and   diffuse   infiltration   of stroma by lymphocytes and histiocytes. What disease corresponds with the described picture?

Explanation

Myocarditis is inflammation of the heart myocardium. Microvascular plethora and diffuse stroma infiltration with lymphocytes and histiocytes are indicative of an inflammatory process taking place in the heart. Heart failure, dilated heart cavities are complications of myocarditis.

141.

A specimen  stained  by Ozheshko method  contains   rod-like   microorganisms stained blue with round terminal components stained  red.  What  are  these components called?

Explanation

Ozheshko – Spores; Burri gins – Capsule
142.

A 38 year old patient with full-blown jaundice,  small cutaneous hemorrhages, general weakness and loss of appetite underwent puncture biopsy  of  liver.  Histological  examination revealed  disseminated  dystrophy,  hepatocyte necrosis, Councilman’s  bodies.   Lobule   periphery has signs of significant infiltration  by lymphocytes, there are also individual multinuclear  hepatocytes.  What   is  the most probable diagnosis?

 

Explanation

          The morphologic changes in acute viral hepatitis are virtually the same regardless of the causative agent and can be mimicked by drug reactions. Grossly, the liver is slightly enlarged. Histologically, the major finding is hepatocellular injury characterized by necrosis of scattered hepatocytes; isolated liver cells or small cell clusters appear as eosinophilic rounded-up cells (apoptotic bodies, councilman’s bodies). Degenerated hepatocytes may also appear ballooned. Macrophages may phagocytize the necrotic hepatocytes and may accumulate clumps of lymphocytes and macrophages. Confluent necrosis may lead to bridging necrosis connecting portal-to-portal, central-to-central or portal-to-central regions of adjacent lobules, signifying a more severe form of acute hepatitis.

143.

A  man  with an injury  of the  dorsal area of his neck was admitted to the resuscitation department. What  muscle occupies this area?

Explanation

Trapezius is a flat, triangular muscle, whose base is facing the spinous processes of the vertebrae. It resides in the upper part of the back and the occiput. The right and left trapezius muscles together form a trapezoid.

Origin: the external occipital protuberance, the superior nuchal line, the ligamentum nuchae and the spinous processes of vertebrae C7 - T12.

Insertion: the spine of the scapula, the acromion and the lateral third of the clavicle.

Action: adducts, rotates, elevates and depresses the scapula.

Nuchal region – neck region

144. In course  of an experiment a toad’s right labyrinth  was destroyed. It will cause amyotonia of the following muscles:

Explanation

Vestibular nuclei receive impulses concerned with muscle tone and posture from vestibular apparatus and cerebellum. Vestibular nuclei in turn convey the impulses to different parts of the body through the anterior and lateral vestibulospinal tracts. Vestibulospinal tracts are concerned with adjustment of position of head and body during angular and linear acceleration; maintenance of muscle tone and posture; position of head and body during acceleration. Therefore, increase transmission of impulse through this tract leads to increase tone. Extensor muscles are also for balance (connected to vestibular apparatus). The inputs from the otolith organs project mainly to the lateral vestibular nucleus, which in turn sends axons in the lateral vestibulospinal tract to the spinal cord. The input from this tract exerts a powerful excitatory influence on the extensor (antigravity) muscles. When hair cells in the otolith organ are activated, signals reach the medial part of the ventral (anterior, motor) horn. By activating the ipsilateral (same side) pool of motor neurons innervating extensor muscles in the trunk and limbs, this pathway mediates balance and the maintenance of upright posture.

145.

A 20 year old patient died from intoxication 8 days  after  artificial  illegal abortion    performed    in    her    14-15th week    of   pregnancy.    Autopsy    of   the corpse revealed yellowish colour of eye sclera   and   of  skin,  necrotic   suppurative endometritis, multiple pulmonary abscesses,  spleen  hyperplasia with  a  big number    of   neutrophils  in   its   sinuses. What complication after abortion was developed?

 

Explanation

Septicopyemia is a form of sepsis, main attributes of which are purulent processes in the entrance of infection and bacterial embolism with formation of abscesses in many organs and tissues. At the dissection, there is primary septic focus, it is usually in the entrance of infection with purulent lymphangitis and lymphadenitis. At first metastatic abscesses appear in the lungs, then in the liver, kidneys (apostematous nephritis), spleen is septic, hyperplastic processes in blood – creating lymphatic tissues.

146.

A patient complains  of dizziness and hearing loss. What nerve is damaged?

Explanation

The receptor areas of the vestibular labyrinth are represented with the following structures:

·        Macula of Utricle

·        Macula of Saccule

·        Ampullary crests

The first two detect linear motion, while the ampullary crest detect angular motion. The vestibulocochlear nerve supply these receptors. Both maculae consist of the sensory hair cells covered with jelly-like susbstance. The substance contains the crystals of calcium carbonate called otoliths (vestibular otolith). The utricle and saccule detect linear movement, also contributing to balance. The ampullary crests reside within each membranous ampulla. They also comprise the sensory hair cells covered with the same jelly-like substance called the ampullary cupula. The ampullae is the sensory organ in the semicircular canal that sense angular (rotational) acceleration of the head, thereby regulating balance. If vestibulocochlear nerve (CN VIII) supply the receptors, then damage to this nerve can cause hearing loss.
147.

A 6 year old child was delivered to a hospital.  Examination revealed   that  the child  couldn’t  fix  his  eyes,  didn’t  keep his  eyes  on  toys,  eye  ground   had   the cherry-red spot sign. Laboratory analyses showed that brain, liver and spleen had high rate  of ganglioside  glycometide.  What congenital  disease is the child ill with?

Explanation

Tay- sachs disease is a progressive neurodegeneration, developmental delay, “cherry-red” spot on macula, lysosomes with onion skin, no hepatosplenomegaly. It is an autosomal recessive disease, caused by the deficiency of hexosaminidase A resulting in the accumulation of GM2 ganglioside in organs of the body (e.g. brain, liver, spleen).

148.

A  patient has  a  cluster  of  matted together dense  lymph nodes  on his neck. Histological  examination of a removed lymph node revealed  proliferation of reticular cells, presence of Reed-Sternberg cells. What disease is meant?

Explanation

        Hodgkin’s disease or lymphogranulomatosis is a disorder involving primarily the lymphoid tissue. It is separated from non-hodgkin lymphoma because:

First, it is characterized morphologically by the presence of distinctive neoplastic giant cells called Reed-sternberg’s cells, admixed with a variable inflammatory infiltrate.

Secondly, it is often associated with distinctive clinical features, including systemic manifestations such as fever.

Reed-sternberg cell: classically, it is a large cell, most often binucleate or bilobed, with two halves often appearing as mirror images of each other. Its identification is essential for the histologic diagnosis. The origin of Hodgkin’s lymphoma is unknown.

149.

While palpating  mammary gland of a patient a  doctor  revealed  an  induration in form  of a node  in the  inferior  medial quadrant. Metastases may  extend  to  the following lymph nodes:

Explanation

Metastases of breast cancer is either local or distant. Local metastases is usually to the lymphatic nodes of breast base, axilla, subclavicular, parasternal nodes. Distant metastases are hematogenic ones, 40-50% to the bones, lungs, and liver.

Cancer of any of the following will metastasize locally through:

            Submandibular salivary gland – submandibular nodes

            Thyroid gland – deep anterior cervical nodes

            Stomach – celiac nodes

            Lungs – visceral thoracic lymph nodes

The dominant lymphatic drainage from the right upper lobe flowed into the superior mediastinal nodes. The sternal or parasternal nodes is one of the superior mediastinal nodes.

150.

A  pregnant woman  was  registered in an antenatal clinic and underwent complex examination for a number of infections.  Blood  serum  contained IgM to  the  rubella  virus.  What  is this  result indicative of?

Explanation

High IgM level usually indicate an acute or primary infection because, it is the first antibody to appear in response to initial exposure to an antigen. Secondary or reinfection show an increase in IgG. IgM antibodies appear early in the course of an infection, this makes it useful in the diagnosis of infectious diseases. Demonstrating IgM antibodies in a patient’s serum indicates recent infection or in a neonate’s serum, it indicates intrauterine infection.

151. Parents of a 3 year old child have been  giving him antibiotics  with purpose of preventing enteric  infections  for a long time.  A  month  later  the  child’s  condition changed  for the  worse.  Blood  examination    revealed    apparent   leukopenia and  granulocytopenia. What  is the  most probable mechanism  of blood changes?

Explanation

152.

A pathological process in bronchi resulted   in   epithelium   desquamation. What  cells will regenerate bronchial  epithelium?

Explanation

The mucosa layer of the bronchi is composed of pseudostratified epithelium with the same cellular composition as the trachea. The basal cells serve as a reserve cell population (regenerative property) that maintains individual cell replacement in the epithelium. Basal cells tend to be prominent because their nuclei form a row in close proximity to the basal lamina. Although nuclei of other cells reside at this same general level within the epithelium, they are relatively sparse. Thus, most of the nuclei near the basement membrane belong to basal cells.

153.

A  patient  ill  with  tuberculosis  died from progressing cardiopulmonary decompensation. Autopsy in the  area  of the right lung apex revealed  a cavity 5 cm in  diameter  communicating with  lumen of  a  segmental   bronchus.  On  the  inside cavity walls are  covered  with caseous masses   with   epithelioid  and   Langhans cells beneath them.  What  morphological form of tuberculosis is it?

 

Explanation

 Secondary tuberculosis usually results from reactivation of dormant, endogenous tubercle bacilli in a sensitized patient who has had previous contact with the tubercle bacillus. Reactivation typically begins in the apical or posterior segments (often 1st and 2nd segments) of one or both upper lobes (“simon’s foci”), where the organisms were seeded during the primary infection. There are 8 forms or stages of the secondary tuberculosis: Acute local tuberculosis; Fibrous local tuberculosis; Infiltrative tuberculosis; Tuberculoma; Caseous pneumonia; Acute cavernous tuberculosis; Fibrous cavernous tuberculosis; Cirrhotic tuberculosis.

Fibrous cavernous tuberculosis is the most frequent form. Macroscopically, the lesions are spherical and cavitary (cavity can contain blood and blood clots); the so-called coin lesions. Microscopically, the outer wall of cavity shows fibrosis or sclerosis. Internal surface may be connected with bronchus. The wall of cavern has 3 membranes:

·        Internal membrane occurs by necrotic tissue

·        Middle membrane occurs by special granular tissue

·        External membrane occurs by connective fibrous tissue

Acute cavernous tuberculosis develops due to lyses of caseous necrosis and characterized by formation of the round cavity.

      Tuberculoma consists of focus necrosis surrounded by fibrous capsule. Size of tuberculoma may be near 2-5cm.
154.

Workers of  a  hothouse farm  work under  conditions of unfavourable microclimate:   air   temperature  is  +37oC , relative  humidity  is 90%, air speed  is 0,2 m/s. The way of heat emission under  these conditions will be:

Explanation

Evaporation is a way the body dissipates heat to the environment by its evaporation via sweat or evaporation of moisture from the skin and respiratory tract mucous membranes of (“wet” heat loss). Evaporation closely related to relative humidity.

Heat Radiation is a way the surface of the human body emits heat to the environment in the form of infrared rays. The amount of heat the body radiates to the environment is proportional to the surface of radiation area and to the difference between the mean values of skin and environment temperature. The surface radiation area is the total surface area of body parts that contact the air. Elimination of heat by radiation increases with a decrease in ambient temperature and decreases with its increase. It is possible to reduce elimination of heat by radiation via reduction of the surface of radiation area (“winding oneself into a ball”). Heat radiation does not require a medium for transfer of heat. (Key words: naked or lightly clothed).

Convection is a way the body eliminates heat by means of transferring heat via moving particles of air or water. To dissipate heat by means of convection, body surface shall be airflowed at a temperature that is lower than the temperature of the skin. At that, air layer contacting with the skin warms up, decreases its density, rises and is replaced by cooler, denser air. By increasing the speed of the air flow (wind, ventilation) heat emission increases significantly as well (forced convection). Convection requires convection current; current of gases or liquids (Key words: air over exposed area of skin).

Conduction is a way the body eliminates heat by means of direct contact with another object. Heat is transferred down the temperature gradient (i.e. from the object of higher temperature to the object of lower temperature). Conduction requires contact with another object (Key words:  in water).

155.

A gynaecologist  was examining  a patient and revealed  symptoms  of genital tract  inflammation. A  smear  from  vagina contains  pyriform  protozoa with a spine, flagella at their  front;  there  is also an undulating membrane. What  disease  can be suspected?

Explanation

In the urogenital tract, the flagellate protozoa, Trichomonas vaginalis is the important pathogen. Trichomonas vaginalis causes trichomoniasis. It is a pear-shaped organism with a central nucleus and four anterior flagella. It has an undulating membrane that extends about two-thirds of its length. It exists only as a trophozoite; there is no cyst form. The organism is transmitted by sexual contact and the primary locations of the organism are the vagina and the prostate. In women, a watery, foul-smelling, greenish vaginal discharge accompanied by itching and burning occurs.

156. A boy has I (I 0I 0) blood  group  and his sister has IV (I A I B ) blood group. What blood groups do their parents have?

Explanation

           IAIO                  IBIO

   IAIB   IAIO        IBIO        IOIO

This is the only combination that can produce a group I (IOIO) and a group IV (IAIB) blood groups.

157.

A patient ill with enteritis accompanied by massive diarrhea has low water rate in the extracellular space, high water rate inside  the  cells and  low blood  osmolarity. What is such disturbance of water-electrolytic  metabolism called?

 

Explanation

       Hypoosmolar hypohydration (salt deficit) develops due to diarrhea, vomiting, sweating, adrenal insufficiency, hypokalemia, CNS lesions, salt-losing nephritis.

          Hyperosmolar hypohydration (water defcit) develops due to the loss of the fluid which lacks electrolytes e.g. in diabetes insipidus, hyperventilation.

          Isoosmolar hypohydration (isoosmotic loss): proportional volume decrease of fluids and electrolytes e.g. in blood loss, burns, diuretic therapy, especially diarrhea in cholera.

         Hypoosmolar hyperhydration (water excess) develops in acute renal failure (in the stage of anuria), Parhon’s syndrome (syndrome of inappropriate ADH secretion –SIADH).

         Hyperosmolar hyperhydration (salt excess) develops rarely e.g. in drinking salt (sea) water, infusion of hypertonic saline, adrenal hyperactivity or steroid therapy.

    Low water level extracellularly – Hypohydration

    Low blood osmolarity – Hypoosmolar           

                Hypoosmolar hypohydration

158.

For the purpose of myocardium infarction treatment a patient was injected with  embryonal stem  cells derived  from this very patient by means  of therapeutic cloning . What transplantation type is it?

Explanation

Allotransplantation: from an individual (donor) to another

Autotransplantation: from one part of the body to another (e.g. skin on thigh to         face).

Xenotransplantation and Heterotransplantation: from one specie to another (e.g. pig to human)

Explantation: removal of a body tissue/organ

159. A histological  specimen  of spleen shows  a vessel  with  a wall consisting  of endothelium and subendothelial layer, median  membrane is absent,  exterior membrane inosculates with the  layers  of spleen connective  tissue. What vessel is it?

Explanation

The tunics of veins are not as distinct or well defined as the tunics of arteries. Veins are divided into 3 types:

* Small veins/venules: postcapillary and muscular venules

* Medium veins

* Large veins

Arteries:

·        Large or elastic arteries

·        Medium or muscular arteries

·        Small arteries and arterioles

Muscular venules are distinguished from postcapillary venules by the presence of a tunica media. Postcapillary venules possess an endothelial lining with its basal lamina and pericytes. Postcapillary venules have no true tunica media.

·        Tunica intima: consists of endothelium with its basal lamina

·        Tunica media: smooth muscle cells

·        Tunica adventitia: collagen fibers

A vessel without tunica media, also lack muscular tissue. Muscular artery, arteriole and artery of mixed type all have tunica media. Only capillaries and postcapillary venules lack tunica media.

Also a prominent internal elastic membrane helps to distinguish muscular arteries from elastic arteries and muscular venules.

160. In course of a conditional experiment the   development  of  mesenchyma  cells was  completely   inhibited.   Development of the following muscular tissue will be disturbed:

Explanation

Mesenchyme refers to loosely organized embryonic connective tissue regardless of origin. Undifferentiated embryonic mesenchymal cells are round/cuboidal in shape. During development, visceral myogenesis is shortly preceded by mesenchymal cell elongation. Undifferentiated embryonic mesenchymal cells from intestine (abundant visceral muscle), lung (some visceral muscle) or kidney (no visceral muscle); these cells differentiate into smooth muscle upon elongation.

161. A  patient  ill  with  chronic   gastritis  went  for  endogastric pH-metry that allowed   to  reveal   decreased  acidity   of gastric juice. It is indicative  of diminished function  of the following cells:

Explanation

Parietal (oxyntic) cells are found in the neck of the fundic glands, among the mucus neck cells and in the deeper part of the gland. They secrete HCl and intrinsic factor. When examined with the transmission electron microscope they are seen to have an extensive intracellular canalicular system that communicates with the lumen of the gland. Also an elaborate tubulo-vesicular membrane system is present in the cytoplasm adjacent to the canaliculi. Numerous mitochondria with complex cristae and many matrix granules supply the high levels of energy necessary for acid secretion.

162.

Vagi   of   an   experimental  animal were  cut  on  both  sides.  What  respiration changes will be observed?

Explanation

163.

A   70  year   old   man   is  ill  with vascular atherosclerosis of lower extremities  and  coronary heart  disease.  Examination    revealed    disturbance  of   lipidic blood composition. The main factor of atherosclerosis pathogenesis is the excess of the following lipoproteins:

Explanation

Atherosclerosis is a multifactorial disease that affects the intima of elastic arteries. The disease is characterized by intramural deposits of lipids, proliferation of vascular smooth muscle cells and fibroblasts and accumulation of macrophages. The risk is correlated with elevated low density lipoprotein (LDL), formed from the catabolism of very low density lipoprotein (VLDL) to a cholesterol ester-protein core that carries some 70% of the total serum cholesterol. Atheroma is specifically associated with high blood LDL levels (as well as total cholesterol levels). Risks is inversely related to the HDL levels, perhaps because high density lipoprotein (HDL) helps clear cholesterol from vessels.

HDL – Good cholesterol

LDL – Bad cholesterol

164.

On the 2-3rd day after stomach resection intestinal peristalsis  wasn’t restored. What  is to be administered for stimulation of gastrointestinal tract?

Explanation

Proserin (neostigmine) is an indirect acting cholinergic agonist (anticholinesterase). It preserves endogenous acetylcholine which can stimulate a greater number of acetylcholine receptors at the muscle endplate. Thereby increasing intestinal peristalsis and tone of urinary bladder.

 

165.

A cardiac electric stimulator was implanted  to  a  75  year   old  man   with heart  rate of 40 bpm. Thereafter the heart rate  rose  up  to 70 bpm.  The  electric  stimulator has  undertaken the  function  of the following heart part:

 

Explanation

For the heart to beat at 40bpm, it means that the SA node is no longer functional in that heart (the AV node has taken over the function of SA node; normal AV node discharge impulses at 40-60bpm). A cardiac electric stimulator that was implanted that discharge impulses at the rate of 70bpm will now take over the function of Sinoatrial (SA) node that beats normally within the range of 60-80bpm (60-80 beats per minute).

166.

A stomatologist injected a patient with a certain  drug  in order  to reduce  salivation during tooth filling. What drug is it?

Explanation

Acetylcholine is a neurotransmitter. It increases salivary secretion and stimulates intestinal secretions and motility. It increases bronchial secretions, increases tone of detrusor urinae muscle, causing expulsion of urine, stimulates ciliary muscle contraction (accommodation). It increases exocrine gland secretions (e.g. lacrimal, salivary, gastric acid); gut peristalsis, bladder contraction, bronchoconstriction, papillary sphincter muscle contraction (miosis). Poisoning by cholinomimetics causes diarrhea, urination, miosis, bronchospasm, bradycardia, excitation of skeletal muscle and central nervous system, lacrimation, sweating and salivation. Cholinomimetics are cholinergic agonists which produce analogous effects of acetylcholine.

Atropine sulfate (M-cholinoblocker) blocks the acetylcholine response.
167.

A  30 year  old  woman  has  applied a  lipstick   with  a  fluorescent  substance for a long time. Then she got a limited erythema and  slight  peeling  on  her  lip border, later  there   appeared transversal striae  and cracks. Special methods  of microscopic examination of the affected area helped  to reveal sensibilized  lymphocytes and macrophages in the connective tissue; cytolysis. What type of immunological hypersensitivity was developed?

Explanation

Type IV (cell mediated, delayed, cellular cytotoxicity): antibody-independent T-cell mediated reactions e.g. positive mantoux reaction (tuberculin test), hashimoto’s thyroiditis or transplant rejection etc.

168.

A  patient ill with  essential  arterial hypertension had a hypertensic crisis that resulted in an  attack  of cardiac  asthma. What is the leading mechanism  of cardiac insufficiency in this case?

Explanation

Hypertension of any etiology is characterized by an increased blood pressure. Of which, the heart must overcome this pressure each time it wants to pump blood out. This increased blood pressure in essential hypertension and hypertensive crisis leads to an increase in workload on the heart, increasing the normal pressure the heart is meant to pump against (↑ afterload). This increase in workload, overtime, can lead to cardiac insufficiency as a result of the excessively increased blood pressure.

169.

A   man   who   has   been   taking   a drug  for  a long  time  cannot  withhold  it because this causes impairment of psychic, somatic  and  vegetative  functions.  Name the syndrome of different disturbances caused by drug discontinuation:

Explanation

Abstinence syndrome is renewal or deterioration of the disease after cessation of the drug administration. It is the withdrawal symptoms that occur after abstinence/stoppage of a drug, especially after taking the drug for some relatively long time (e.g. a narcotic).

       deterioration of the disease: 'impairment of psychic...'

170. A patient consulted  a stomatologist about  purulent inflammation of his gums. What  drug will be the  most effective  if it  is suspected that a causative agent is an anaerobe?

Explanation

Metronidazole forms toxic free radical metabolites in the bacterial cell that damage DNA. It is bactericidal, and an antiprotozoal. It is used to treat Giardia, Entamoeba, Trichomonas, Gardnerella vaginalis, Anaerobes (bacteroides, Clostridium difficile). It can be used with a proton pump inhibitor and clarithromycin for “triple therapy” against Helicobacter pylori.

171. Rats    being    under     stress    have muscular hypertonia and high arterial pressure, high glucose concentration in blood and intensified secretion of corticotropin  and   corticosteroids.  In   what stress phase are these animals?

Explanation

Antishock phase of shock is when the compensatory mechanisms of the body are activated to antagonize the devastating effects of shock. Shock is characterized by decrease in blood pressure, which leads to decrease blood circulation, but in the antishock phase, stress hormones such as corticosteroids and catecholamines (epinephrine, norepinephrine) and the whole sympathoadrenal system are fully active to prevent decrease in blood pressure and other characteristics of shock. Epinephrine, cortisol (glucocorticoids) have the capacity to increase blood pressure, increase glucose concentration; increase respiration (ventilation) etc. the antishock phase is a component of the compensatory stage of shock.

172.

A patient came to the hospital complaining about quick fatigability and apparent  muscle   weakness.   Examination revealed  an autoimmune disease  that causes disorder of functional receptor condition   in   neuromuscular   synapses. What transmitter will be blocked?

Explanation

Myasthenia gravis is an autoimmune disease of neuromuscular junction caused by antibodies to cholinergic receptors. It is characterized by grave weakness of the muscle due to the inability of neuromuscular junction to transmit impulses from nerve to the muscle. It is caused due to the development of auto-antibodies (IgG auto-antibodies) against the receptors of acetylcholine. That is, the body develops antibodies against its own acetylcholine receptors. These antibodies prevent binding of acetylcholine with its receptors or destroy the receptors. So, though the acetylcholine release is normal, it cannot execute its action.

173.

Examination of a 35 year old patient revealed high acidity of gastric juice. What receptors should  be  blocked  in order  to reduce  it?

Explanation

Gastric acid secretion by parietal cells of the gastric mucosa is stimulated by acetylcholine, histamine and gastrin. Histamine binding causes activation of adenylyl cyclase, whereas binding of prostaglandin E2 inhibits this enzyme. Gastrin and acetylcholine acts by inducing an increase in intracellular calcium levels (it is evident that blocking acetylcholine receptor wouldn’t stop acid secretion, because gastrin can do the same work with the same mechanism). Therefore, it is more effective to block Histamine receptors. Antagonists of the histamine H2 receptor, blocks the action of histamine at all H2 receptors but their chief clinical use is to inhibit gastric acid secretion, being particularly effective against nocturnal acid secretion. H2-receptor blockers include cimetidine (Tagamet), ranitidine, famotidine, nizatidine. NB: H1-receptor blocker – loratadine (for allergic reactions).

174. Examination of a young woman revealed a node-like, soft and elastic homogenous tumour of pinkish-white colour along the acoustic nerve. The tumour contains   cell  bundles   with  oval nuclei. Cellular fibrous bundles form rhythmic  structures made  up  by parallel rows of regularly  oriented cells arranged in form of a palisade with cell-free homogenous zone (Verocay bodies) between them. What tumour is it?

Explanation

Neurinoma or Schwannoma is a benign tumor of peripheral nervous system. It is formed of spinder-like cells with rod-shaped nuclei. The cells and fibers form rhythmical structures. An acoustic neurinoma is a benign tumor that may develop on the hearing and balance nerves (CNVIII vestibulocochlear nerve) near the inner ear. The tumor results from an overproduction of Schwann cells – small sheet-like cells that normally wrap around nerve fibers like onion skin and help support the nerves.

175.

A  45  year  old  woman   is  ill  with breast  cancer. Her left arm has symptoms of  lymphatic  system  insufficiency  - limb edema,  lymph  node  enlargement. What form of lymphatic circulation  insufficiency is it?

Explanation

        Types of lymph-circulatory insufficiency:

·        Mechanical insufficiency

·        Dynamic insufficiency

·        Insufficiency of absorption/Resorption insufficiency

Mechanical insufficiency is present where lymph flow is hindered by a mechanical factor. The mechanical factor may be of an anatomical, organic nature e.g. occlusion of lymphatics by obstructive lymphangitis, thrombosis of the lymphatics (e.g. during cancer metastasis), filariasis.

The other two types has to do with a disbalance of proteins that upsets the filtration and absorption pressures.

176. An   endocrinal  gland   with parenchyma consisting of epithelium and neural tissue  is  under   morphological examination. Epithelial trabecules have two types of cells: chromophilic and chromophobic. Identify this organ:  

Explanation

The pituitary gland (hypophysis) is composed of glandular epithelial tissue and neural (secretory) tissue. The hypophysis consists of two major subdivisions; the adenohypophysis and the neurohypophysis. The adenohypophysis is further subdivided into the pars distalis (anterior lobe), pars tuberalis and pars intermedia. The neurohypophysis is divided into the pars nervosa (infundibular process), infundibulum and tuber cinereum of the median eminence. The pars distalis contains two main cell types, chromophobe cells and chromophil cells. The chromophils (chromophilic cells) are subdivided into acidophils (alpha cells) and basophils (beta cells).

177. While  performing an inguinal  canal operation on account  of hernia  a surgeon damaged the  canal’s contents. What exactly was damaged?

Explanation

         What forms the walls of the ingunal canal:

·        Anteriorly: the aponeuroses of the external and internal oblique muscles.

·        Posteriorly: the transversalis fascia.

·        Superiorly: the arching fibers of the internal oblique and transverses abdominis muscles.

·        Inferiorly: the inguinal ligament.

Contents:

·        Men: Spermatic cord (funiculus spermaticus) and the ilioinguinal nerve

·        Women: round ligament of the uterus (ligamentum teres uteri) and the ilioinguinal nerve.

      Since the gender was not specified in the question, then it means the question s referring to inguinal canal content in men, which is the Funiculus spermaticus.

178. A  12 year  old  child has  intolerance to some foodstuffs. Their consumption causes an allergic reaction in form of itching  skin  eruptions.  What   antihistaminic drug should be administered so that the child could attend school?

Explanation

        Loratadine is an antihistamine (antiallergic drug) which refers to the classic H1-receptor blocker. These compounds do not influence the formation or release of histamine. Rather, they block the receptor-mediated response of a target tissue. They are much more effective in preventing symptoms than reversing them once they have occurred. These agents, desloratadine, fexofenadine, and loratadine show the least sedation i.e. least effect on CNS (they don’t induce sleep).

179.

A  patient  that   entered  the  admission  office  had  the  following  signs  of acute cardiac insufficiency: paleness, acrocyanosis,   frequent shallow  respiration. What drug is indicated in this case?

 

Explanation

The symptoms describe cardiac insufficiency. Cardiac glycoside is the most appropriate in this case. Cardiac glycosides are glycosidic drugs of plant origin having cardiac inotropic property, increase myocardial contractility and output. Corglyconum is short acting for acute cardiac insufficiency and administered intravenously.

Digitoxin is long-acting but basically administered orally and used mainly for chronic heart failure (also digoxin).

Cordiamin – analeptic; Adrenaline is commonly used in shocks and coma; 

180. A  26 year  old  man  is in the  torpid shock phase  as a result  of a car accident. In blood:  3, 2 · 109/l. What  is the  leading mechanism  of leukopenia development?

Explanation

Shock is a form of stress which is accompanied by the release of stress hormones (catecholamines, cortisol). Stress-induced changes in blood leukocyte distribution may represent an adaptive response. This represents a redistribution of leukocytes from the blood to other organs such as the skin, draining sentinel lymph nodes and other compartments. Such leukocyte redistribution may enhance immune function in compartments to which immune cells traffic during stress.

181.

A  patient had  hemorrhagic stroke. Blood examination revealed strengthened kinin concentration. The patient was prescribed contrical.  It was administered in  order  to  inhibit  the  following  proteinase:

Explanation

α2-macroglobulin and α1-antitrypsin are protease inhibitors which can inhibit the pancreatic proteases and prevent further tissue protein breakdown. 

182. A patient with bacterial pneumonia was  prescribed  benzylpenicillin. What  is the mechanism  of its antibacterial effect?

Explanation

          Penicillins  are β-lactam antibiotics that inhibit the synthesis of bacterial cell walls and are considered bactericidal too. They bind penicillin binding proteins (transpeptidases) and block transpeptidase cross-linking of peptidoglycan in cell wall. They also activate autolytic enzymes. Benzylpenicillin is a short acting penicillin.

183. Hartnup disease is caused by point mutation of only one gene which results in  disturbance  of  tryptophane  absorption in the bowels and its resorption in the renal tubules.  It is the reason  for disorder of both  digestive  and  urination systems. What genetic phenomenon is observed  in this case?

Explanation

* Pleiotropy: one gene defect contributes to multiple phenotypic effects → disorder of connective tissue and eye lens structure, abnormalities of CNS, arachnodactylia. Marfan is FBN1 gene mutation on chromosome 15; Marfan and Hartnup is one gene defect causing multiple phenotypic defects; likewise Hartnup disease (digestive and Urinary systems)

* Codominance: both alleles contribute to the phenotype of the heterozygote. e.g. blood group AB

184.

A  patient who  has  been  treated  in a  neural   clinic  and  has  been   taking   a sedative  for  a long  time  got  the  following complication: cough, rhinitis, epiphora. What drug caused these disturbances?

Explanation

        Doses of bromides have to be determined individually. The bromides are excreted from the organism rather slowly (50-60days), so they may be accumulated and cause the signs of chronic poisoning (bromism). The irritative action of bromides leads to inflammation of the mucous membranes which is accompanied by cough, rhinitis, conjunctivitis and diarrhea.

185.

In  order  to  accelerate healing  of  a radiation ulcer  a vitamin  drug was administered. What drug is it?

Explanation

Vitamin A and the amino acid glutamine help to regenerate healthy epithelial cells. Vitamin A helps to form and maintain healthy skin, mucous membrane and teeth. Several studies, such as the one published in the June 2008 edition of the “Journal of vascular nursing”, have shown reduced levels of vitamin A in patients suffering from leg ulcers. Therefore, due to its function in maintaining healthy skin, then it can help in hastening the healing of radiation ulcer.
186.

Inoculum from  pharynx  of a patient ill with angina was inoculated into blood-tellurite  agar.  It  resulted  in  growth   of grey, radially striated (in form of rosettes) colonies 4-5 mm in diameter. Gram-positive  bacilli  with  clublike  thickenings on their ends placed in form of spread  wide apart  fingers are visible by microscope. What microorganisms are these?

 

Explanation

Diphtheria bacteria (Corynebacterium diphtheria) is Gram positive, pleomorphic, often club-shaped rods and are arranged in palisades or in V (at an angle) or L-shaped formations. Media used for isolation are Tellurite agar & Lὄffler medium. Lὄffler nutrient medium consists of coagulated serum & nutrient broth. Selective indicator medium containing tellurite are used in selective culturing. K tellurite is used to inhibit the accompanying flora.

187. Plasmic factors  of blood  coagulation are  exposed  to post-translational modification  with  the  participation of  vitamin K .  It  is  necessary   as  a  cofactor   in  the enzyme   system   of   γ-carboxylation  of protein factors  of blood  coagulation due to the increased  affinity of their molecules with calcium ions. What amino acid is carboxylated in these proteins?

Explanation

Vitamin K is essential for the formation of various clotting factors in the liver, namely clotting factor II, VII, IX and X; Protein C and S.

Factor II – Prothrombin

Factor VII – Stable factor

Factor IX – Christmas factor

Factor X – Stuart-Prower factor

These factors undergo vitamin K-dependent post-translational modification, whereby a number of their glutamic acid residues are carboxylated to form ɣ-carboxyglutamic acid residues. The ɣ-carboxyglutamyl residues bind calcium ions which are essential for interaction between the coagulation factors and platelet membranes. This oral anticoagulants block epoxide reductase and creation of active form of vitamin K resulting in disturbances in prothrombin and proconvertin synthesis in liver.

Deficiency of vitamin K → ↓clotting factors → prolongation of coagulation time → hemorrhages.

188. Pharmacological effects of anti-depressants are connected with inhibition of an enzyme  catalyzing  biogenic  amines noradrenaline and  serotonine in the  mitochondrions of  cerebral neurons. What enzyme participates in this process?

Explanation

Norepinephrine (noradrenaline), epinephrine (adrenaline) and dopamine are catecholamines produced in chromaffin cells of adrenal medulla from tyrosine. The catecholamines are inactivated by oxidative deamination catalyzed by monoamine oxidase (MAO) and by O-methylation carried out by Catechol-O-methyltransferase. The metabolic products of these reactions are excreted in the urine as vanillymandelic acid from epinephrine and norepinephrine; and homovanillic acid from dopamine.

189.

An  oncological  patient was prescribed methotrexate. With the lapse of time target  cells of the tumour lost susceptibility to this drug. There  is change  of gene expression  of the following enzyme:

Explanation

Methotrexate is structurally related to folic acid and acts as an antagonist of that vitamin by inhibiting dihydrofolate reductase (in humans), which is the enzyme that converts folic acid to its active coenzyme form, tetrahydrofolic acid. Trimethoprim and pyrimethamine inhibit the same enzyme but in bacteria and protozoa respectively. This decreases dTMP, needed for DNA synthesis.

190.

As  a  result   of  prophylactic  medical examination a 7 year old boy was diagnosed  with Lesch-Nyhan syndrome (only   boys  fall  ill  with  it).  The   boy’s parents are  healthy   but  his  grandfather by his mother’s side suffers from the same disease.  What  type of disease  inheritance is it?

Explanation

Autosomal dominant: often due to defects in structural genes. Many generations, both male and female are affected. Found in every generation; no generation is left out (skipped). Parent – child in every generation.

Autosomal recessive is usually seen in some generations (other generations are skipped).

It is sex linked, if only males or only females(X-linked) are affected.

["only boys fall ill..."]
191.

Planned mass vaccination  of all newborn 5-7 day old children  against tuberculosis plays an important role in tuberculosis prevention. In  this  case  the following vaccine is applied:

Explanation

 

Tuberculin (Mantoux) skin test: this test is done by intradermal injection of tuberculoprotein (tuberculin), purified protein derivative (PPD). Type IV hypersensitivity reaction.

Immunization against tuberculosis is induced by injection of attenuated strains of bovine type of tubercle bacilli, Bacilli Calmette Guerin (BCG).

192. A patient with hypertensic crisis was admitted to the cardiological  department, he was injected intravenously with an anti-hypertensive drug - salt of an alkaline-earth metal. What drug was injected?

Explanation

 

193. In course  of a small pelvis operation it  became   necessary   to  ligate  an  ovarian  artery.  What  formation may  be  accidentally ligated together with it?

Explanation

     The ureter connects the renal pelvis with the urinary bladder. It is 30cm long, 5-6mm in diameter and lies extraperitoneally (i.e. devoid of peritoneal investment). Each ureter has 3 parts: abdominal, pelvic and intramural parts. Entering the pelvic inlet, the right ureter crosses the right external iliac artery and the left crosses the left common iliac artery. In females, the ureters run posterior to the broad ligament of uterus and then along the free border of ovary. Laterally to the cervix of uterus, the ureters loop around the inferior border of the broad ligament cross the uterine vessels at a right angle and pass between the anterior wall of vagina and the urinary bladder to reach the fornix of the latter posteriorly.

194.

A patient with acute  morphine poisoning  was delivered to a hospital.  What specific narcotic  antagonist should be chosen in this case?

Explanation

Acute poisoning with narcotic analgesics (e.g. morphine) is characterized by sleep or unconsciousness, normal or increased reflexes, normal muscle tone, miosis, bradycardia, cheyne-stokes breath, the retention of urine, a spasm of intestine and bowel. This condition can be treated with:

·        Stomach lavage with 0.5% solution of potassium permanganate.

·        Intravenous injection of Naloxone (naloxone is an antagonist of narcotic analgesic).

·        Atropine; for decrease in the vagal action of morphine.

195.

The  upper   lobe  of  the  right  lung is enlarged, grey  and  airless,  the  inscision surface  is dripping  with turbid  liquid, the  pleura  has many  fibrinogenous films; microscopical  examination of alveoles revealed   exudate  containing  neutrophils,  desquamated alveolocytes   and  fibrin fibers. The bronchus  wall is intact. What is the most probable diagnosis?

Explanation

196.

A   28  year   old   patient  had   high arterial   pressure,  hematuria   and   facial  edemata. In  spite  of  treatment renal insufficiency  was  progressing.   6  months later  the  patient died  from  uremia.  Microscopic examination of his kidneys  and their glomerules  revealed  proliferation of capsule  nephrothelium and  of podocytes with  \\\"demilune\\\" formation, sclerosis  and hyalinosis of glomerules. What disease corresponds with the described  picture?

Explanation

We have acute, subacute and chronic glomerulonephritis. Subacute glomerulonephritis does not necessarily follow acute attacks. It is usually characterized by the general symptoms of glomerulonephritis like proteinuria, hematuria, edema and azotemia persisting for many days and even weeks. Renal changes include those of rapidly progressive and membranoproliferative glomerulonephritis.    DEMILUNE”

197.

A  patient  with  a  knife  wound   in the  left lumbar  part  was delivered  to the emergency  hospital.  In course  of operation  a surgeon  found  that  internal organs were  not  damaged but  the  knife  injured one   of  muscles   of  renal   pelvis.  What muscle is it?

 

Explanation

The anterior group of muscles of the pelvic girdle comprises the flexors, which includes the iliopsoas (iliacus and psoas major muscles) and psoas minor muscle. The iliopsoas consists of two muscles – the psoas major and iliacus. The psoas major is a thick, elongated fusiform muscle, which extends from the lumbar region downward, occupying the space between the bodies of the vertebrae and their transverse processes. The muscle becomes narrower below, and passing under the inguinal ligament, it fuses with the iliacus muscle. The psoas major is a major component of the renal bed together with other muscles such as the diaphragm, quadratus lumborum and transverse abdominis muscle.

198. A  man  with  a  stab  wound  in  the area  of quadrilateral foramen  applied  to a doctor.  Examination revealed  that  the patient was unable  to  draw  his arm  aside  from  his  body.  What  nerve  is  most probably  damaged?

Explanation

The axillary nerve (nervus axillaris) from brachial plexus, is the greatest branch of the short branches of brachial plexus. It arises from the posterior cord and proceeds to the quadrangular/quadrilateral foramen. It supplies the deltoid and teres minor muscles. The axillary nerve and posterior circumflex humeral artery pass through the quadrangular space. Borders of the quadrangular space:

·        Superior: Teres minor

·        Inferior: Teres major

·        Medial: Long head of triceps

·        Lateral: humerus

199.

Autopsy of a man ill with severe hypothyroidism  revealed   that   connective tissue, organ stroma, adipose and cartilaginous tissues  were  swollen,  semi- transparent, mucus-like.  Microscopic examination of  tissues  revealed   stellate cells  having  processes  with  mucus between them.  What  type  of  dystrophy is it?

Explanation

Stromal vascular carbohydrate degenerations develop due to disturbance of glycosaminoglycans and glycoproteids metabolism. When glycoproteid metabolism is disturbed, chromotropic substances are released from the protein bonds. They accumulate in the main substance of the connective tissue. Collagen fibers change into mucus-like mass. Connective tissue mucin is associated with:

·        Mucoid or myxoid degeneration in some tumors (myxomas)

·        Neurofibromas, soft tissue sarcomas etc

·        Myxomatous change in the dermis in myxedema

·        Myxoid change in the synovium in ganglion on the wrist.

The condition results in colliquative necrosis with formation of cavities filled with mucus.

200.

Ultramicroscopical examination of \"dark\" hepatocyte  population in  the  cell cytoplasm  detected a developed granular endoplasmic reticulum. What function has this organella in these cells?

Explanation

     Granular or rough endoplasmic reticulum (i.e. rough surface) is a region of endoplasmic reticulum associated with ribosomes. It is the site of protein synthesis and modification of newly synthesized proteins.